You are on page 1of 78

INTEGRATED REVIEW 2: Advanced Financial Accounting and Reporting (AFAR)

#1 | Partnership Accounting: Nature, Formation, Operations, Dissolutions, Liquidation

1. The JPB partnership reported net income of P160,000 for the year ended December 31,
20x4. According to the partnership agreement, partnership profits and losses are to be
distributed as follows:

J P B
Salaries P 50,000 P60,000 P30,000
Bonus on net income 10% 5% 10%
Remainder (if positive) 60% 30% 10%
Remainder (if negative) 30% 40% 30%

How should partnership net income for 20x4 be allocated to J, P, and B?


J P B
A. P96,000 P48,000 P16,000
B. P58,000 P64,000 P38,000
C. P60,000 P60,000 P40,000
D. P66,000 P68,000 P46,000

(RESA, May 2018)


2. DO is admitted into the partnership of RE and MI by investing cash equivalent to ¼ of their
capital. Which of the following is true after the admission of DO?
A. Assets of the partnership will increase
B. Total partners’ equity remain the same
C. RE and MI capital decreased by ¼
D. Assets of the partnership will remain the same

(RESA, May 2018)


For numbers 3-4, refer to the problem below:
The following condensed balance sheet is presented at February 18, 2018 for the
partnership of Dana and Janis, who share profits and losses in ratio of 60:40, respectfully.
Cash P150,000 Accounts payable P120,000
Non-cash assets 300,000 Dana, Capital 195,000
Dana, Loan 20,000 Janis, Capital 155,000

The non-cash assets realized P250,000 in actual liquidation


3. How much would Dana receive if cash is distributed to the partners just before the start of
actual liquidation?
A. P 5,000
B. P 18,000
C. P 30,000
D. P 0
4. How much cash would Janis receive upon final liquidation, assuming no prior cash
distribution had been made to the partners.
A. P 135,000
B. P 145,000
C. P 100,000
D. P 0
(PRTC, May 2018)

5. PP contributed P24,000 and CC contributed P48,000 to form a partnership, and they agreed
to share profits in the ratio of their original capital contributions. During the first year of
operations, they made a profit of P16,290; PP withdrew P5,050 and CC P8,000. At the start
of the following year, they agreed to admit GG into the partnership. He was to receive a
one-fourth interest in the capital and profits upon payment of P30,000 to PP and CC, whose
capital accounts were to be reduced by transfers to GG’s capital account of amounts
sufficient to bring them back to their original capital ratio.

How should the P30,000 paid by GG be divided between PP and CC?


A. PP, P 9,825; CC, P 20,175
B. PP, P 15,000; CC, P 15,000
C. PP, P 10,000; CC, P 20,000
D. PP, P 9,300; CC, P 20,700
(Dayag, 2015)

6. Scott, Joe, and Ed are liquidating their partnership. At the date the liquidation begins Scott,
Joe, and Ed have capital account balances of P162,000, P192,500, and P215,000,
respectively and the partners share profits and losses 40%, 35%, and 25%, respectively. In
addition, the partnership has a P36,000 Notes Payable to Scott and a P20,000 Notes
Receivable from Ed. When the liquidation begins, what is the loss absorption power with
respect to Joe?
A. P192, 500
B. P 67,375
C. P550,000
D. P770,000
(Dayag, 2015)
7. Which of the following is not considered a legitimate expense of a partnership?
A. Interest paid to partners based on the amount of invested capital
B. Depreciation on assets contributed to the partnership by partners
C. Salaries for management hired to run the business
D. Supplies used in the partners’ offices
(Punzalan, 2014)
8. In the AA-BB partnership, AA and BB had a capital ratio of 3:1 and a profit and loss
ratio of 2:1 respectively. The bonus method was used to record CC’s admittance as a new
partner. What ratio would be used to allocate, to AA and BB, the excess of CC’s
contribution over the amount credited to CC’s capital account?
A. AA and BB’s new relative ratio.
B. AA and BB’s new relative profit and loss ratio.
C. AA and BB’s old capital ratio.
D. AA and BB’s old profit and loss ratio.

(Dayag 2013)
9. The following is the priority sequence in which liquidation proceeds will be distributed
for a partnership:
A. Partnership drawings, partnership liabilities, partnership loans, partnership capital
balances
B. Partnership liabilities, partnership loans, partnership capital balances.
C. Partnership liabilities, partnership loans, partnership drawings, partnership capital
balances.
D. Partnership liabilities, partnership capital balances, partnership loans

(Punzalan, 2014)
10. Partnership capital and drawings accounts are similar to the corporate
A. Paid in capital, retained earnings, and dividends accounts.
B. Retained earnings accounts
C. Paid in capital and retained earnings accounts
D. Preferred and common stock accounts.
(Punzalan, 2014)

11. An advantage of the partnership as a form of business organization would be


A. Partners do not pay income taxes on their share in partnership income.
B. A partnership is bound by the act of the partners.
C. A partnership is created by mere agreements of the partners.
D. A partnership may be terminated by the death or withdrawal of a partner.
(Punzalan, 2016)

12. In the liquidation of a partnership it is necessary to (1) distribute cash to the partners; (2)
sell non-cash assets; (3) allocate any gain or loss on realization to the partners; and (4) pay
liabilities. These steps should be performed in the following order
A. 2,3,4,1
B. 2,3,1,4
C. 3,2,1,4
D. 3,2,4,1
(Punzalan, 2016)

13. It is the change in the relation of the partners caused by any partner ceasing to be associated
in the carrying on of the business.
A. Dissolution
B. Liquidation
C. Incorporation
D. Break-up
(Millan, 2017)
14. On January 1, 2016, Atta and Boy agreed to form a partnership contributing their
respective assets and equities subject to adjustment. On that date, the following were
provided:

Atta Boy
Cash 28,000 62,000
Accounts receivable 200,000 600,000
Inventories 120,000 200,000
Land 600,000
Building 500,000
Furniture and Fixtures 50,000 35,000
Intangible assets 2,000 3,000
Accounts Payable 180,000 250,000
Other liabilities 200,000 350,000
Capital 620,000 800,000

The ff adjustments were agreed upon:


A. Accounts receivable of P 20,000 and P 40,000 are uncollectible in A’s and B’s
respective books.
B. Inventories of P 6,000 and P 7,000 are worthless in A’s and B’s respective books
C. Intangible assets are to be written off in both books.
What will be the capital balances of the partners after adjustments?
Atta Boy
A. 592,000 750,00
B. 600,000 700,00
C. 592,000 756,300
D. 600,000 750,000
(Punzalan, 2016)

15. Partner Ae first contributed P50,000 of capital into existing partnership on March 1, 2016.
On June 1, 2016, said partner contributed another P20,000. On September 1, 2016, he
withdrew P15,000 from the partnership. Withdrawal in excess of P10,000 are charged to
partner’s capital accounts. What is the annual weighted average capital balance of Partner
Ae?
A. 32,500
B. 51,667
C. 60,000
D. 48,333
(Punzalan, 2016)

16. Maxwell is trying to decide whether to accept a salary of P 40,000 or salary of P 25,000
plus a bonus of 10% of net income after salaries and bonus as a means of allocating profit
among partners. Salaries traceable to the other partners are estimated to be P 100,000.
What amount of income would be necessary so that Maxwell would consider choices to
be equal
A. 165,000
B. 290,000
C. 265,000
D. 305,000
(Punzalan, 2016)

For numbers 17 and 18 refer to the problem below:


On June 30, 2016, the condensed balance sheet for the partnership of Eddy, Fox, and
Grimm together with their respective profit and loss sharing percentage, were as follows:

Assets, net of liabilities P 320,000

Eddy, Capital (50%) P 160,000


Fox, Capital (30%) 96,000
Grimm, Capital (20%) 64,000
Total Capital P 320,000
17. Eddy decided to retire from the partnership by mutual agreement is to be paid P 180,000
out of partnership funds for his interest. Total goodwill implicit in the agreement is to be
recorded. After Eddy’s retirement, what will be capital balances of the other partners?
Fox Grimm
A. 84,000 56,000
B. 102,000 68,000
C. 108,000 72,000
D. 120,000 80,000
(Punzalan, 2016)

18. Assume instead that Eddy remains in the partnership and that Hamm is admitted as a new
partner with 25% interest in the capital of the new partnership for a cash payment
P140,000. Total goodwill implicit in the transaction is to be recorded. Immediately after
admission of Hamm, Eddy’s capital account balance should be

A. 280,000
B. 210,000
C. 160,000
D. 140,000
(Punzalan, 2016)
For numbers 9 and 10 refer to the problem below:
The ABC Partnership has assets with book value of P240,000 and a market value of
P195,000, outside liabilities of P70,000, loans payable to Partner Able of P20,000, and
capital balances for Partners Able, Baker and Chapman of P70,000, P30,000 and P50,000,
respectively. The partners share profits and losses equally.

19. How would the first P100,000 of available assets be distributed?


A. P70,000 to outside liabilities, P20,000 to able and balances equally among partners
B. P70,000 to outside liabilities, and P30,000 to Able
C. P70,000 to outside liabilities, P25,000 to Able, and P5,000 to Chapman
D. P40,000 to Able, P20,000 to Chapman, and the balance equally among partners
(Punzalan, 2017)

20. If all outside creditors and loans to partners had been paid. How would the balance of the
assets be distributed assuming Chapman had already received assets with a value of
P30,000?
A. Each of the partners would receive P30,000
B. Each of the partners would receive P40,000
C. Able: P70,000; Baker: P30,000; Chapman: P20,000
D. Able: P55,000; Baker: P15,000; Chapman: P5,000
(Punzalan, 2017)

21. If a partner’s capital balance is credited for an amount greater than or less than the fair
value of his net contribution, the excess or deficiency is called a
A. Bonus
B. Goodwill
C. Discount
D. Premium
(Millan, 2016)

22. Before allocation of loss, which of the following items are allocated first?
A. Salaries
B. Bonuses to partners
C. Interest on the capital of an industrial partner
D. All of these
(Millan, 2016)

23. After the admission of a new partner, the total partnership capital increased by the fair
value of the new partner’s net contributions to the partnership. The admission was
accounted for
A. Under the goodwill method
B. Under the bonus method
C. As a purchase of interest
D. As an investment in the partnership
(Millan, 2016)

24. On May 1, 2016, Cobb and Mott formed a partnership and agreed to share profits and losses
in the ratio of 3:7, respectively. Cobb contributed a parcel of land that cost him P10,000.
Mott contributed P40,000 cash. The land was sold for P18,000 on May 1, 2016,
immediately after formation of the partnership. What amount should be recorded in
Cobbs’s capital account on formation of the partnership?
A. 18,000
B. 17,400
C. 15,000
D. 10,000
(Punzalan, 2018)

For numbers 5 to 6:
The Grey and Redd Partnership was formed on January 2, 2016. Under the partnership agreement,
each partner has an equal initial capital balance. Partnership net income or loss is allocated 60%
to Grey and 40% to Redd. To form the partnership, Grey initially contributed assets costing
P30,000 with a fair value of P60,000 on January 2, 2016, and Redd contributed P20,000 cash.
Drawings by the partners during 2016 totaled P3,000 by Grey and P9,000 by Redd. The partnership
net income in 2016 was P25,000.
(Punzalan 2018)

25. Under the goodwill method, what is Redd’s initial capital balance in the partnership?
A. 20,000
B. 25,000
C. 40,000
D. 60,000

26. Under the bonus method, what is the amount of bonus?


A. 20,000 bonus to Grey
B. 20,000 bonus to Redd
C. 40,000 bonus to Grey
D. 40,000 bonus to Redd

27. If a partnership has net income of P44,000 and Partner X is to be allocated bonus of 10%
of income after the bonus, what is the amount of bonus Partner X will receive?
A. 3,000
B. 3,300
C. 4,000
D. 4,400
(Punzalan, 2018)

28. A partnership has the following accounting amounts:


Sales P 700,000
Cost of goods sold 400,000
Operating expenses 100,000
Salary allocations to partners 130,000
Interest paid to banks 20,000
Partners' drawings 80,000
What is the partnership net income (loss)?
A. 200,000
B. 180,000
C. 50,000
D. (30,000)
(Punzalan, 2018)
Solution:

29. Ranken purchases 50% of Lark’s capital interest in the K and L partnership for P22,000. If
the capital balances of Kim and Lark are P40,000 and P30,000, respectively, Ranken’s
capital balance following the purchase is
A. 22,000
B. 35,000
C. 20,000
D. 15,000
(Punzalan, 2018)

30. The following condensed balance sheet is presented for the partnership of Smith and Jones,
who share profits and losses in the ratio of 60:40, respectively:
Other assets P 450,000
Smith, loan 20,000
P 470,000

Accounts payable P120,000


Smith, capital 195,000
Jones, capital 155,000
P 470,000

The partners decided to liquidate the partnership. If the other assets are sold for P385,000,
what amount of the available cash should be distributed to Smith?
A. 136,000
B. 156,000
C. 159,000
D. 195,000
(Punzalan, 2018)

31. Flat and Iron partnership agreement provides for Flat to receive 20% bonus on profits
before bonus. Remaining profits and losses are divided between Flat and Iron in the ratio
2:3, respectiviely. Ehich partner has greater advantage when the partnership has a profit or
when it has a loss
A. Profit: Flat; Loss:Iron
B. Profit:Flat; Loss: Flat
C. Profit: Iron; Loss: Flat
D. Profit: Iron; Loss: Iron
(Punzalan, 2018)
32. During 2016, Young and Zinc maintained average capital balances in their partnership of
160000 and 100000, respectively. The partners receive 10% interest on average capital
balances and residual profit or loss is divided equally. Partnership profit before interest was
4000. By what amount should Zinc’s capital account change for the year?
A. 11000 decrease
B. 2000 increase
C. 1000 decrease
D. 12000 increase
(Punzalan, 2018)

Mitz, Marc and Mart are partners sharing profits in the ratio of 5:3:2, respectively. As of
December 31, 2016, their capital balances were 95,000 for Mitz, 80000 for Marc & 60000 for
Mart. On Jan 1, 2017, the partners admitted Vince as a new partner and according to their
agreement, Vince will contribute 80000 in cash to the partnership and also pay 10000 for 15%
of Marc’s share. Vince will be given a 20% share in profits, while the original partners’ share
will be approximately the same as before. After the admission of Vince, the total capital will
be 330000 and Vince’s Capital will be 70000

33. The total amount of goodwill to the old partners, upon the admission of Vince would be:
A. 7000
B. 15000
C. 22000
D. 37000

34. . The balance of Marc’s Capital, after admission of Vince would be:
A. 72600
B. 74600
C. 79100
D. 81100
(Punzalan, 2018)

35. As of Dec 31, the books of AME Partnership showed capital balances of A - 40,000; M
25,000; E-5,000. The partners’ profit or loss ratio is 3:2:1, respectively. The partners
decided to dissolve and liquidate. They sold all the non-cash assets for 37,000 cash. After
settlement of all liabilities amounting to 12,000, they still have 28,000 cash left for
distribution. The loss on realization for distribution is
A. 40,000
B. 42,000
C. 44,000
D. 45,000
(Punzalan, 2018)
36. In installment liquidation, which of the following statements is correct regarding the
partial settlement of the partners’ claims?

A. The claims of the partners and outside creditors are partially settled in proportion
B. No distribution is made to the partners until after all non cash assets are realized
C. The carrying amount of unsold non cash assets is treated as loss
D. Estimates of future liquidation costs do not affect the distribution to the partners
(Milan, 2016)
37. Under the entity theory, a partnership is

A. Viewed as having its own existence apart from the partners


B. Viewed through the eyes of the partnera
C. A separate legal and tax entity
D. Unable to enter into contracts in its own name
(FT&C)

38. Which of the following statements is true concerning the treatment of salaries in
partnership accounting?

A. The salary of a partner is treated in the same manner as salaries of corporate


employees
B. Partner salaries are equal to the annual partner draw
C. Partner salaries may be used to allocate profits and losses; they are not
considered expenses of the partnership
D. Partners salaries are directly closed to the capital account
(FT&C)

39. Which of the following is true?

A. A stipulation that excludes one or more partners from any share in the profits or
losses is valid
B. The income summary account is credited in the entry to record the distribution of
profits
C. In the absence of any agreement, salary allowances to partners shall be provided
when the operations yield losses
D. Salary and interest allowances are reported in the statement of comprehensive
income as salaries and interest expense

40. Partners C & K share profits and losses equally after each has been credited in all
circumstances with annual salary allowances of 15,000 & 12,000, respectively. Under this
arrangement, C will benefit by 3,000 more than K in which of the following:

1. Only if the partnership has earnings of 27,000 or more for the year
2. Only If the partnership does not incur a loss for the year
3. In all earnings or loss situation
4. Only if the partnership has earnings of at least 3000 for the year
(FT&C 11e)
41. On June 30, 2015, the balance sheet of Western Marketing, a partnership, is summarized
as follows:
Sundry assets…………………………………………………………….P150,000
West, Capital…………………………………………………………….…90,000
Tern, Capital………………………………………………………………. 60,000
Wes and Tern share profit and losses at a 60:40 ratio, respectively. They agreed to take in
Cuba as a new partner, who purchases 1/8 interest of West and Tern for P25,000. What is
the amount of Cuba’s capital to be taken up in the partnership books if the book value
method is used?
A. P12,500
B. P18,750
C. P25,000
D. P31,250
(Dayag, 2015)

42. In the AD partnership, Allen’s capital is P140,000 and Daniel’s is P40,000 and they share
income in a 3:1 ratio, respectively. They decide to admit David to the partnership.

Allen and Daniel agree that some of the inventory is obsolete. The inventory account is
decreased before David is admitted. David invests P40,000 for a one-fifth interest. What is
the amount of inventory written down?
A. P4,000
B. P10,000
C. P15,000
D. P20,000
(Dayag, 2015)

43. In the AD partnership, Allen’s capital is P140,000 and Daniel’s is P40,000 and they share
income in a 3:1 ratio, respectively. They decide to admit David to the partnership. David
directly purchases one-fifth interest by paying Allen P34,000 and Daniel P10,000. The land
account is increased before David is admitted. By what amount is the land account
increased?

A. P40,000
B. P36,000
C. P20,000
D. P10,000
(Dayag, 2015)

44. RR and XX formed a partnership and agreed to divide initial capital equally, even though
RR contributed P25,000 and XX contributed P21,000 in identifiable assets. Under the
bonus method approach to adjust the capital accounts, XX’s unidentifiable assets should
be debited for:

A. P11,500
B. P4,000
C. P2,000
D. 0
(Dayag, 2015)

45. Partner A first contributed P50,000 of capital into an existing partnership on March 1, 2015.
On June 1, 2015, the partner contributed another P20,000. On September 1, 2015, the
partner withdrew P15,000 from the partnership. Withdrawals in excess of P10,000 are
charged to the partner’s capital account. The annual weighted-average capital balance is

A. P62,000
B. P51,667
C. P60,000
D. P48,333
(Dayag, 2015)
46. For financial accounting purposes, assets of an individual partner contributed to a
partnership are recorded by the partner at

A. Historical cost
B. Book value
C. Fair market value
D. Lower of cost or market
(Dayag, 2015)

47. Which of the following interest component calculation bases is the least susceptible to
manipulation when allocating profits and losses to partners?

A. Beginning capital account balance


B. Average of beginning and ending capital balance
C. Weighted average capital account balance
D. Ending capital balance
(Dayag, 2015)

48. In a partnership, interest on capital investment is accounted for as a(n)

A. Return on investment
B. Expense
C. Allocation of net income
D. Reduction of capital
(Dayag, 2015)

49. What is the underlying purpose of the interest on capital balances component of allocating
partnership profits and losses?

A. Compensate partners who contribute economic resources to the partnership


B. Reward labor and expertise contributions
C. Reward for special responsibilities taken
D. None of the above
(Dayag, 2015)

50. What is the underlying purpose of the salary component of allocating partnership profits
and losses?

A. Compensate partners who contribute economic resources to the partnership


B. Reward labor and expertise contributions
C. Reward for special responsibilities taken
D. None of the above
(Dayag, 2015)
51. A, B, and C are partners in an accounting firm. Their capital account balances at year-end
were A P90,000; B P110,000; C P50,000. They share profits and losses on a 4:4:2 ratio,
after the following special terms:
• Partner C is to receive a bonus of 10% of net income after the bonus.
• Interests of 10% shall be paid on that portion of a partner’s capital in excess of
P100,000
• Salaries of P10,000 and P12,000 shall be paid to partners A & C respectively.
Assuming a net income of P44,000 for the year, the total profit share of Partner C was:
A. P7,800
B. P16,800
C. P19,400
D. P19,800
(Dayag, 2015)

52. X and Y are in partnership, sharing profits equally and preparing their accounts to 31
December each year. On 1 July 2015, Z joined in the partnership, and from that date profits
are shared X 40%, Y 40%, and Z 20%.

In the year ended 31 December 2015, profits were:


6 months to 31 June 2015 P200,000
6 months to 31 December 2015 P300,000

It was agreed that X and Y only should bear equally the expense for a bad debt of P40,000
written-off in the six months to 31 December 2015 in arriving at the P300,000 profit.

Which of the following correctly states X’s profit share for the year?
A. P216,000
B. P200,000
C. P220,000
D. P224,000
(Dayag, 2015)

53. Pol and Loc are partners with capitals of P200,000 and P100,000 and sharing profits and
losses 3:1 respectively. They agree to admit Chic as partner. Chic invests P125,000 for a
25% interest in the firm. Parties agree that the total firm capital after Chic’s admission is
to be P425,000.
The capital balances of the partners after Chic’s admission are:
A. Pol, P214,062.50; Loc, P104,687.50; and Chic, P106,250
B. Pol, P200,000.00; Loc, P100,000.00; and Chic, P125,000
C. Pol, P239,062.50; Loc, P 79,687.50; and Chic, P125,000
D. Pol, P250,000.00; Loc, P125,000.00; and Chic, P100,000
(Guerrero, 2013)

54. Michelle, an active partner in the Michelle-Esme partnership receives an annual bonus of
25% of the partnership income after deducting the bonus. For the year ended, December
31, 2013, partnership income before the bonus amounted to P240,000. The bonus of
Michelle for the year 2013 is:
A. P45,000
B. P48,000
C. P80,000
D. P60,000
(Guerrero, 2013)

55. Rita, Sisa, and Tina are partners with the capital balances on June30, 2013 of P60,000,
P60,000 and P40,000, respectively. Profits and losses are shared equally. Tina withdraws
from the partnership. The partners agree that Tina is to take certain furniture at their
secondhand value of P2,400 and cash for the balance of her interest. The furniture is carried
on the books as fully depreciated.

The amount of cash to be paid to Tina and the capital balances of the remaining partners
after the retirement of Tina are:
Cash Rita, capital Sisa, capital
A. P40,000 P60,000 P60,000
B. P37,000 P61,200 P61,200
C. P38,400 P60,800 P60,800
D. P42,800 P58,800 P58,800
(Guerrero, 2013)

For question 55-56 refer to the problem below


The AA, BB, CC Partnership was formed on January 2. 2019. The original cash
investments were as follows:
AA P 48,000
BB P 72,000
CC P108,000
According to the general partnership contract, the partners were to be remunerated as
follows:
a. Salaries of P72,000 for AA, P6,000 for BB, and P6,800 for CC.
b. Interest at 12% on the average capital account balances during the year.
c. Remainder divided 40% to AA, 30% to BB, and 30% for CC.
Income before partners’ salaries for the year ended December 31, 2019, was P46,040. AA
invested an additional P12,000 in the partnership on July 1; CC withdrew P18,000 from
the partnership on October 1, and, as authorized by the partnership contract, AA, BB, and
CC each withdrew P375 monthly against their shares of net income for the year.

56. Determine the share of partner AA in the net income:


A. P18,416
B. P17,616
C. P13,080
D. P5,880
(ReSA, 2018)

57. Determine the capital balance of partner CC on December 31, 2019:


A. P108,770
B. P104,270
C. P100,112
D. P99,312
(ReSA, 2018)

58. If the salaries to partners are to be recognized as operating expenses by the partnership,
what amount is the share of partner BB in the net income?
A. P18,416
B. P14,190
C. P8,190
D. P7,812
(ReSA, 2018)

(For question 59 – 60)


DD and EE was organized and began operations of March 1, 2019. On that date, DD
invested P75,000 and EE invested land and building with current fair value of P40,000 and
P50,000, respectively. EE also invested P30,000 in the partnership on November 1, 2019
because of its shortage of cash. The partnership contract includes the following
remuneration plan:
DD EE
Annual Salary P9,000 P12,000
Annual interest on average capital account balance 10% 10%
Remainder 60% 40%

The annual salary was to be withdrawn by each partner in 12 monthly installments. During
fiscal year ended, February 28, 2020, DD and EE had net sales of P25,000, cost of goods
sold of P140,000 and total operating expenses of P50,000 (excluding partners’ salaries and
interest on average capital account balances). Each partner made monthly cash drawings
in accordance with partnership contract.

59. Determine the share of partner DD in the net income:


A. P29,400
B. P33,000
C. P36,000
D. P23,400
(ReSA, 2018)

60. The capital balance of each partner on March 1, 2020 should be:
A. DD P95,400; EE P138,600
B. DD P66,000; EE P82,000
C. DD P108,000; EE P147,000
D. DD P99,000; EE P135,000
(ReSA, 2018)

61. Which of the following is not considered a legitimate expense of a partnership?


a. Supplies used in the partners’ office
b. Depreciation on assets contributed to the partnership by partners
c. Salaries for management hired to run the business
d. Interest paid to partners based on the amount of invested capital

Millan Textbook (2016)

62. If the partnership agreement does not specify how income is to be allocated, profits and
loss should be allocated
A. Equally.
B. In proportion to the weighted average of capital invested during the period.
C. Equitably so that partners are compensated for the time and effort expended on
behalf of the partnership.
D. In accordance with their capital contributions.
(Millan, 2016)
63. When Mill retired from the partnership of Mill, Yale, and Lear, the final settlement of
Mill’s interest exceeded Mill’s capital balance. Under the bonus method, the excess
A. Was recorded as goodwill.
B. Was recorded as an expense.
C. Reduced the capital balances of Yale and Lear.
D. Had no effect on the capital balances of Yale and Lear.
(Millan, 2016)
64. State the correct order of the claims on the personal assets of a partner,
I. The partner’s separate creditors
II. To the other partner’s by way of contribution
III. The partnership creditors
A. I, III, II
B. I, II, III
C. III, II, I
D. II, I, III
(Millan, 2016)
65. It is the change in the relation of the partners caused by any partner ceasing to be associated
in the carrying on of the business.
A. Dissolution
B. Liquidation
C. Incorporation
D. Break-up
(Millan, 2016)
66. MM, NN, and OO are partners with capital balances on December 31, 2012 of P300,000,
P300,000 and P200,000, respectively. Profits are shared equally. OO wishes to withdraw
and it is agreed that OO is to take certain equipment with second-hand value of P50,000
and a note for the balance of OO’s interest. The equipment are carried on the books at
P65,000. Brand new equipment may cost P80,000. Compute for: (1) OO’s acquisition of
the second-hand equipment will result to reduction in capital; (2) the value of the note that
will OO get from the partnership’s liquidation.
A. (1) P 15,000 each for MM and NN, (2) P150,000
B. (1) P5,000 each for MM, NN and OO, (2) P145,000
C. (1) P5,000 each for MM, NN and OO, (2) P195,000
D. (1) P7,500 each for MM and NN, (2) P145,000
(Dayag, 2013)
67. The partnership agreement of XX, YY and ZZ provides for the year-end allocation of net
income in the following order:
• First, XX is to receive 10% of net income up to P200,000 and 20% over
P200,000.
• Second, YY and ZZ each are to receive 5% of the remaining income over
P300,000.
• The balance of income is to be allocated equally among the three partners.
The partnership’s 2011 net income was P500,000 before any allocations to partners. What
amount should be allocated to XX?
A. P202,000
B. P216,000
C. P206,000
D. P220,000
(Dayag, 2013)
68. RR and XX formed a partnership and agreed to divide initial capital equally, even though
RR contributed P25,000 and XX contributed P21,000 in identifiable assets. Under the
bonus approach to adjust the capital accounts. XX’s unidentifiable assets should be debited
for:
A. P11,500
B. P 4,000
C. P 2,000
D. P 0
(Dayag, 2013)
69. As of December 31, 2012, the books of Ton Partnership showed capital balances of: T,
P40,000; O, P25,000; N, P5,000. The partner’s profit and loss ratio was 3:2:1, respectively.
The partners decided to liquidate and they sold all non-cash assets for P37,000. After
settlement of all liabilities amounting P12,000, they still have cash of P28,000 left for
distribution. Assuming that any capital debit balance is uncollectible, the share of T in the
distribution of the P28,000 cash would be:
A. P17,800
B. P18,000
C. P19,000
D. P17,000
(Dayag, 2013)
70. CC, PP and AA, accountants, agree to form a partnership and to share profits in the ratio
of 5:3:2. They also agreed that AA is to be allowed a salary of P28,000, and that PP is to
be guaranteed P21,000 as his share of the profits. During the first year of operation, income
from fees are P180,000, while expenses total P96,000. What amount of net income should
be credited to each partner’s capital account?
A. CC, P28,000, PP, P16,800, AA, P11,200
B. CC, P25,000, PP, P21,000, AA, P38,000
C. CC, P24,000, PP, P22,000, AA, P38,000
D. CC, P25,000, PP, P21,000, AA, P39,000
(Dayag, 2013)
71. Allen retired from the partnership of Allen, Beck and Chale. Allen’s cash settlement from
the partnership was based on new goodwill determined at the date of retirement plus the
carrying amount of the other net assets. As a consequence of the settlement, the capital
accounts of Beck and Chale were decreased. In accounting for Allen’s withdrawal, the
partnership could have used the:

BONUS METHOD GOODWILL METHOD


A. No Yes
B. No No
C. Yes Yes
D. Yes No
(Milan, 2016)

72. Which of the following has the least priority of payment in case of partnership
liquidation?

A. Priority claims such as artisans. Government, liquidation expenses


B. Secured creditors to the extent of covered by the proceeds from the sale of pledged
assets.
C. Unsecured credit to the extent covered by proceeds from sale of unpledged (or free)
assets.
D. The partners’ capital balances.
(Milan 2016)

73. State the proper order of liquidation


I Outside creditors
II Owners’ interests
III Inside creditors
A. I, III, II
B. I, II, III
C. III, II, I
D. II, I, III

(Milan, 2016)

74. According to the Philippine Civil Code, if only the shares of each partner in the profits
has been agreed upon, the share of each in the losses shall be
A. equally
B. equally, but the industrial partner shall not share in the loss
C. the same as the sharing in profits
D. the same as the sharing in profits. However, the industrial partner shall not
share in the loss.
(Milan, 2016)

75. Which of the following is not considered a legitimate expense of a partnership?


A. Supplies used in the partners’ offices.
B. Depreciation on assets contributed to the partnership by partners.
C. Salaries for management hired to run the business.
D. Interest paid to partners based on the amount of invested capital.
(Milan, 2016)

76. CC, PP and AA, accountants, agree to form a partnership and to share profits in the ratio
of 5:3:2. They also agreed that AA is to be allowed a salary P28,000 and that PP is to be
guaranteed P21,000 as his share of the profits. During the first year of operation, income
from fees are P180, 000, while expenses total P96,000. What amount of net income should
be credited to each partner’s capital account?
A. CC, P28,000 PP, P16,800 AA, P11,200
B. CC, P25,000 PP, P21,000 AA, P38,000
C. CC, P24,000 PP, P22,000 AA, P38,000
D. CC, P25,000 PP, 21, 000 AA, P39, 000
(Dayag, 2015)
77. The same information in Number 32, except the partnership had a loss of P125, 624 after
the interest and salaries to partners, by what amount should BB’s capital account change
increase(decrease)?
A. P (115,443) C. P (41,875)
B. 23,865 D. (18,010)
(Dayag, 2015)

78. PP, QQ and RR, partners to a firm, have capital balances of P11, 200, P13, 000 and P5,
800, respectively, and share profits in the ratio of 4:2:1. Prepare a schedule showing how
available cash will be given to the partners as it becomes available. Who among the
partners shall be paid first with an available cash of P1, 400?
A. QQ B. No One C. RR D. PP

79. The August, Albert and Gerry partnership became insolvent on January 1, 2015, and the
partnership is being liquidate as as practicable. In this respect the following information
for the partners has been marshaled.

Capital Balances Personal Assets Personal Liabilities


August P 70,000 P 80,000 P 40,000
Albert - 60,000 30,000 50,000
Gerry - 30,000 70,000 30,000

Assume that residual profits and losses are shared equally among the three partners.
Based on this information, calculate the maximum amount that August can expect to
receive from the partnership liquidation is:

A. P20, 000 C. P70,000


B. 40,000 D. 110,000
(Dayag, 2015)

80. RR and XX formed a partnership and agreed to divide initial capital equally, even though
RR contributed P25,000 and XX contributed P21,000 in identifiable assets. Under the
bonus approach to adjust the capital accounts. XX’s unidentifiable assets should be debited
for:

A. 11,500 C. 2,000
B. 4,000 D. 0

(Dayag, 2015)

81. Partnership capital and drawing accounts are similar to the corporate

A. Paid-in capital, retained earnings, and dividend accounts.


B. Retained earnings account.
C. Paid-in capital and retained earnings accounts.
D. Preferred and common stock accounts.
(Gleim)

82. The partnership agreement is an express contract among the partners (the owners of the
business). Such an agreement generally does not include

A. A limitation on a partner’s liability to creditors.


B. The rights and duties of the partners.
C. The allocation of income between the partners.
D. The rights and duties of the partners in the event of partnership dissolution.
(Gleim)

83. A partnership records a partner’s investment of assets in the business at

A. The market value of the assets invested.


B. A special value set by the partners.
C. The partner’s book value of the assets invested.
D. Any of the above, depending upon the partnership agreement.
(RPCPA 0598)

84. Assume that C has a P50,000 equity in the partnership of “A, B, and C.” Partner C arranges
to sell his entire interest to D for P80,000 Cash. Partners A and B agree to the admission
of D. At what amount will the equity of the incoming partner, D, be shown in the balance
sheet?

A. at P50,000.
B. at P50,000 and the P30,000 will be divided equally among the original partners.
C. at P80,000
D. at P80,000 and the P30,000 will represent Goodwill which will be apportioned between
E. the existing equities of A and B.
(RPCPA 107)

Albion and Blaze share profits and losses equally. Albion and Blaze receive salary allowances of
$20,000 and $30,000, respectively, and both partners receive 10% interest on their average capital
balances. Average capital balances are calculated at the beginning of each month balance
regardless of when additional capital contributions or permanent withdrawals are made
subsequently within the month. Partners’ drawings are not used in determining the average capital
balances. Total net income for 2006 is $120,000.

Albion Blaze
January 1 capital balances $ 100,000 $ 120,000
Yearly drawings ($1,500 a month) 18,000 18,000
Permanent withdrawals of capital:
June 3 ( 12,000 )
May 2 ( 15,000 )
Additional investments of capital:
July 3 40,000
October 2 50,000

85. What is the weighted-average capital for Albion and Blaze in 2006?

A. $100,000 and $120,000.


B. $105,333 and $126,667.
C. $110,667 and $119,583.
D. $126,667 and $105,333.
(Beams, 2009)
86. If the average capital for Albion and Blaze from the above information is $112,000 and
$119,000, respectively, what will be the total amount of profit allocated after the salary
and interest distributions are completed?

A. $70,000.
B. $73,100.
C. $75,000.
D. $80,000.
(Beams, 2009)

87. If the average capital balances for Albion and Blaze are $100,000 and $120,000,
what will the final profit allocations for Albion and Blaze in 2006?

A. $50,000 and $70,000.


B. $54,000 and $66,000.
C. $70,000 and $50,000.
D. $75,000 and $45,000.
(Beams, 2009)
For number 88 to 89 refer to the problem below:
Bloom and Carnes share profits and losses in a ratio of 2:3, respectively. Bloom and Carnes receive
salary allowances of $10,000 and $20,000, also respectively, and both partners receive 10%
interest based upon the balance in their capital accounts on January 1. Partners’ drawings are not
used in determining the average capital balances. Total net income for 2006 is $60,000. If net
income after deducting the interest and salary allocations is greater than $20,000, Carnes receives
a bonus of 5% of the original amount of net income.

Bloom Carnes
January 1 capital balances $ 200,000 $ 300,000
Yearly drawings ($1,500 a month) 18,000 18,000

88. What are the total amounts for the allocation of interest, salary, and bonus, and, how
much over-allocation is present?

A. $60,000 and $0.


B. $80,000 and $20,000.
C. $83,000 and $0.
D. $83,000 and $23,000.
(Beams, 2009)
89. The XYZ partnership provides a 10% bonus to Partner Y that is based upon
partnership income, after deduction of the bonus. If the partnership's income is
$121,000, how much is Partner Y's bonus allocation?

A. $11,000.
B. $11,450.
C. $11,650.
D. $12,100.
(Beams, 2009)

Lara, Ives, and Jack are in the process of liquidating their partnership. Since it may take several
months to convert the other assets into cash, the partners agree to distribute all available cash
immediately, except for $10,000 that is set aside for contingent expenses. The balance sheet and
residual profit and loss sharing percentages are as follows:

Cash $ 400,000 Accounts payable $ 200,000


Other assets 200,000 Hara, capital (40%) 135,000
Ives, capital (30%) 216,000
Jack, capital (30%) 49,000

Total assets $ 600,000 Total liab./equity $ 600,000

90. How much cash should Ives receive in the first distribution?

A. $146,000.
B. $147,000.
C. $153,000.
D. $156,000
(Beams, 2009)

91. Which of the following transactions shall not affect the capital balance of a partner?
A. Share of a partner in the partnership’s net loss
B. Receipt of bonus by a partner from another partner based on the agreement
C. Advances made by the partnership to a partner
D. Additional investment by a partner to the partnership

CPAR Testbank

92. Which of the following will not result to the dissolution of a partnership?
E. Insolvency of the partnership
F. Admission of a new partner in an existing partnership
G. Assignment of an existing partner’s interest to a third person
H. Retirement of a partner

CPAR Testbank

93. He refers to a partner who contributed not only money and property but also industry
to the newly formed partnership.
A. Industrial partner
B. Nominal partner
C. Capitalist-industrial partner
D. Capitalist partner
CPAR 2017 Pre-Board

94. It refers to a type of partnership wherein all partners are liable to the creditors pro-rata
up to the extent of personal or separate assets after the partnership’s assets are
exhausted.
A. General partnership
B. Partnership by estoppel
C. Limited partnership
D. Particular partnership
CPAR 2017 Pre-Board

95. Which of the following statements concerning the formation of partnership business is
correct?
A. Philippine Financial Reporting Standards (PFRS) allows recognition of goodwill
arising from the formation of partnership.
B. The juridical personality of the partnership arises from the issuance of certification of
registration.
C. The parties may become partners only upon contribution of money or property but not
of industry or service.
D. The capital to be credited to each partner upon formation may not be the amount
actually contributed by each partner.
CPAR 2017 Pre-Board
96. The partners, C and D, share profits 3:2. However, C is to receive a yearly bonus of
20% of the profits, in addition to his profit share. The partnership made a net income
for the year of P960,000 before the bonus. Assuming C’s bonus is computed on profit
after deducting said bonus, how much profit share will D receive?
A. P307,200
B. P320,000
C. P640,000
D. P160,000
CPAR 2017 Pre-Board

97. A, B, and C are partners and share profits and losses as follows: Salaries of P40,000 to
A; P30,000 to B; and none to C. If net income exceeds salaries, then a bonus is allocated
to A. The bonus is 5 percent of net income after deducting salaries and the bonus.
Residual profits or residual losses are allocated 10 percent to A, 20 percent to B, and
70 percent to C. If net income before salaries and bonus is P140,000, how much is the
share of A?
A. P50,150
B. P43,333
C. P46,667
D. P50,000
CPAR 2017 Pre-Board

98. Carson and Lamb establish a partnership to operate and used-furniture business under
the name of C & L Furniture. Carson contributes furniture that cost P60,000 and has a
fair value of P90,000. Lamb contributes P30,000 cash and delivery equipment that cost
P40,000 and has a fair value of P30,000. The partners agree to share profits and losses
60% to Carson and 40% to Lamb. Calculate the peso amount of inequality that will
result if the initial noncash contributions of the partners are recorded at cost rather than
fair market value.
A. P30,000
B. P10,000
C. P20,000
D. P18,000
CPAR 2017 Pre-Board
Items 99 and 100 are based on the following:
Lucy and Annie were partner sharing profits and losses equally. Ochie was admitted as a partner
by contributing cash of P60,000 for one-third interest in the firm. They agreed to set the total
capital at P210,000 after Ochie’s admission. Prior to Ochie’s admission, the old partner’s capital
accounts were Lucy, P48,000, and Annie, P96,000.
99. The capital balance of Annie after Ochie’s admission was
A. P92,667
B. P94,000
C. P91,000
D. P96,000
100. Assuming that Ochie will share one-fourth interest on the partnership assets the
capital balance of Annie after Ochie’s admission is
A. P96,000
B. 99,750
C. P99,000
D. P102,750
CPAR 2017 Pre-Board

101. A partnership agreement calls for allocation of profits and losses by salary
allocations, a bonus allocation, interest on capital, with any remainder to be allocated by
preset ratios. If a partnership has a loss to allocate, generally which of the following
procedures would be applied?

A. Any loss would be allocated equally to all partners.


B. Any salary allocation criteria would not be used.
C. The bonus criteria would not be used.
D. The loss would be allocated using the profit and loss ratios, only.
(Fisher, 2008)

102. Della Reise was admitted to a partnership. She contributed $25,000 cash plus
equipment she purchased for $50,000 and which had accumulated depreciation for tax
purposes of $20,000. The fair value of the equipment was $35,000. She also assumed 1/3
of partnership debt of $15,000. Her beginning capital balance was $48,000. For tax
purposesher partnership interest should be initially valued at

A. $60,000
B. $48,000
C. $55,000
D. $65,000
(Fisher, 2008)
103. Under the bonus method, when a new partner is admitted to the partnership, the
total capital of the new partnership is equal to:

A. the book value of the previous partnership + the fair market value of the
consideration paid to the existing partnership by the incoming partner
B. the book value of the previous partnership + any necessary asset write ups from
book value to market value + the fair market value of the consideration paid to the
existing partnership by the incoming partner
C. the book value of the previous partnership - any asset write downs from book to
market value + the fair market value of the consideration paid to the existing
partnership by the incoming partner
D. the fair market value of the new partnership as implied by the value of the
incoming partner's consideration in exchange for an ownership percentage in the
new partnership
(Fisher, 2008)

104. Assume that a partnership had assets with a book value of $240,000 and a market
value of $195,000, outside liabilities of $70,000, loans payable to partner Able of $20,000,
and capital balances for partners Able, Baker, and Chapman of $70,000, $30,000, and
$50,000. How would the first $100,000 of available assets be distributed assuming profits
and losses are allocated equally?

A. $70,000 to outside liabilities, $20,000 to Able, and the balance equally among the
partners
B. $70,000 to outside liabilities and $30,000 to Able
C. $70,000 to outside liabilities, $25,000 to Able, and $5,000 to Chapman
D. $40,000 to Able, $20,000 to Chapman, and the balance equally among the partners
(Fisher, 2008)

105. Partners Dalton, Edwards, and Finley have capital balances of $40,000, 90,000 and
$30,000, respectively, immediately prior to liquidation. Total remaining assets have a book
value of $160,000, the liabilities having been paid. Among these remaining assets is a
machine with a fair value of $35,000. The partners split profits and losses equally. Edwards
covets the machine and is willing to accept it for $35,000 in lieu of cash. The other partners
have no designs on specific assets, only cash in liquidation. How much cash, in addition to
the machine, would be first distributed to Edwards, before any of the other partners
received anything?

A. $15,000
B. $50,000
C. $166,667
D. $300,000
(Fisher, 2008)

106. Partners Thomas, Adams and Jones have capital balances of $24,000, $45,000, and
$90,000 respectively. They split profits in the ratio of 3:3:4, respectively. Under a
predistribution plan, one of the partners will get the following total amount in liquidation
before any other partners get anything:

A. $22,500
B. $30,000
C. $40,000
D. $75,000
(Fisher, 2008)

107. Which of the following statements are true when comparing corporations and
partnerships?

A. Partnership entities provide for taxes at the same rates used by corporations.
B. In theory, partnerships are more able to attract capital.
C. Like corporations, partnerships have an infinite life.
D. Unlike shareholders, general partners may have liability beyond their capital
balances.

108. Which of the following characteristics of a partnership most likely explains why a
public accounting firm is organized as a partnership from a public policy viewpoint?

A. A partnership is not a taxable entity.


B. A partnership is characterized by unlimited liability.
C. A partnership is characterized by a fiduciary relationship among the partners.
D. Salaries to the partners are not considered a component of net income.
(Fisher, 2008)

109. The partnership agreement is an express contract among the partners (the owners
of the business). Such an agreement generally does not include:

A. A limitation on a partner’s liability to creditors.


B. The rights and duties of the partners.
C. The allocation of income between the partners.
D. The rights and duties of the partners in the event of partnership dissolution.

(Punzalan, 2016)

110. A partnership records a partner’s investment of assets in the business at

A. The market value of the assets invested.


B. A special value set by the partners.
C. The partner’s book value of the assets invested.
D. Any of the above, depending upon the partnership agreement.
(Punzalan, 2016)

111. When property other than cash is invested in a partnership, at what amount should
the noncash property be credited to the contributing partner’s capital account?

A. Fair value at the date of recognition.


B. Contributing partner’s original cost.
C. Assessed valuation for property tax purposes.
D. Contributing partner’s tax basis.

(Punzalan, 2016)

112. X, Y, Z are capitalist partners and D an industrialist partner. The partnership


reported a net loss of P200,000. How much is the share of D?

A. 0
B. 10,000
C. 25,000
D. 100,000
(Punzalan, 2016)

113. Assume that C has a P715,000 equity in the partnership of “A, B, and C.” Partner
C arranges to sell his entire interest to D for P80,000 Cash. Partners A and B agree to the
admission of D.At what amount will the equity of the incoming partner, D, be shown in
the balance sheet?

A. at P715,000.
B. at P50,000 and the P30,000 will be divided equally among the original partners.
C. at P80,000
D. at P80,000 and the P30,000 will represent Goodwill which will be apportioned
between
E. the existing equities of A and B.
(Punzalan, 2016)

114. Partner Morgan is personally insolvent, owing P600,000. Personal assets will only
bring P200,000 when liquidated. At the same time, Morgan has a credit balance in the
partnership of P120,000. The capital amounts of the other partners total a credit balance
of P250,000. Under the doctrine of marshaling of assets, how much the personal creditors
of Morgan can collect?

A. 120,000
B. 200,000
C. 320,000
D. 570,000
(Punzalan, 2016)

For Numbers 115 to 116 refer to the problem below


As of December 31, the books of AME Partnership showed capital balances of A-P40,000; M-
P25,000; And E-P5,000. The partners’ profit and loss ratio were 3:2:1, respectively. The partners
decided to dissolve and liquidate. They sold all the non-cash assets for P37,000 cash. After
settlement of all liabilities amounting to P12,000, they still have P28,000 cash left for distribution.

115. The loss on the realization of the non-cash assets was

A. P40,000
B. P42,000
C. P44,000
D. P45,000

(Punzalan, 2016)

116. Assuming that any partner’s capital debit balance is uncollectible, the share of A in
the P28,000 cash for distribution would be

A. P19,000
B. P18,000
C. P17,800
D. P40,000
(Punzalan, 2016)

117. The following balance sheet is presented for the partnership A, B and C, who share
profits and losses in the respectively ratio of 5:3:2

Assets Liabilities and Capital


Cash Php 120,000 Liabilities Php 280,000
Other Assets 1,080,000 A, Capital 560,000
B, Capital 320,000
C, Capital 40,000
Total Php 1,200,000 Total Php 1,200,000

Assume the three partners decided to liquidate the partnership. If the other assets are sold
for P800,000, how should the available cash be distributed to each partner?
A B C
A. 280,000 320,000 40,000
B. 324,000 236,000 16,000
C. 410,000 230,000 0
D. 412,000 228,000 0

(Punzalan, 2016)

118. Partners Almond, Barney and Colors have capital balances of P20,000, P50,000,
and P90,000, respectively. They split profits in the ratio of 2:4:4, respectively. Under a
safe cash distribution plan, one of the partners will get the following total amount in
liquidation before any other partners get anything
A. 0
B. 15,000
C. 40,000
D. 180,000

(Punzalan, 2016)
119. Methods exist for the division of partnership profits and losses

A. Equally
B. Arbitrary ratio
C. Capital contribution ratio
D. All of the above
(Dayag, 2015)
120. For a partner to withdraw or retire from the partnership, the total interest of a partner
should be properly determined which includes:

A. Share in the profit or loss of the partnership


B. Adjustments in assets and liabilities to reflect fair market values
C. Loans to and from partnership
D. All of the above
(Dayag, 2015)
121. On December 1, 2015, EE and FF formed a partnership, agreeing to share for profits and
losses in the ratio of 2:3, respectively. EE invested a parcel of land that cost him P25,000.
FF invested P30,000 cash. The land was sold for P50,000 on the same date, thress hours
after formation of the partnership. How much should be the capital balance of EE right
after formation?

A. P25,000
B. P30,000
C. P60,000
D. P50,000
(Dayag, 2015)

122. MM, NN, and OO are partners with capital balances on December 31, 2015 of P300,000,
P300,000 and P200,000, respectively. Profits are shared equally. OO wishes to withdraw
and it is agreed that OO is to take certain equipment with second-hand value of P50,000
and a note for the balance of OO’s interest. The equipment are carried on the books at
P65,000. Brand new equipment may cost P80,000. Compute for: (1) OO’s acquisition of
the second-hand equipment will result to reduction in capital; (2) the value of the note that
will OO get from the partnership’s liquidation.

A. (1) P15,000 each for MM and NN, (2) P150,000


B. (1) P5,000 each for MM, NN and OO, (2) P145,000
C. (1) P5,000 each for MM, NN and OO, (2) P195,000
D. (1) P7,500 each for MM and NN, (2) P145,000
(Dayag, 2015)

123. JJ and KK are partners who share profits and losses in the ratio of 60%: 40%, respectively.
JJ’s salary is P60,000 and P30,000 for KK. The partners are also paid interest on their
average capital balances. In 2015, JJ received P30,000 of interest and KK, P12,000. The
profit and loss allocation is determined after deductions for the salary and interest
payments. If KK’s share in the residual income (income after deducting salaries and
interest) was P60,000 in 2015, what was the total partnership income?
A. P192,000
B. P345,000
C. P282,000
D. P387,000
(Dayag, 2015)

124. CC, PP, and AA, accountants, agree to form a partnership and to share profits in the ratio
of 5:3:2. They also agreed that AA is to be allowed a salary of P28,000, and that PP is to
be guaranteed P21,000 as his share of the profits. During the first year of operation, income
from fees are P180,000, while expenses total P96,000. What amount of net income should
be credited to each partner’s capital account?

A. CC, P28,000, PP, P16,800, AA, P11,200


B. CC, P25,000, PP, P21,000, AA, P38,000
C. CC, P24,000, PP, P22,000, AA, P38,000
D. CC, P25,000, PP, P21,000, AA, P39,000
(Dayag, 2015)

125.The following condensed balance sheet is presented for the partnership of AA, BB, and
CC, who share profits and losses in the ratio of 4:3:3, respectively:

Cash P160,000
Other Assets 320,000
Total P480,000

Liabilities P180,000
AA, capital 48,000
BB, capital 216,000
CC, capital 36,000
Total P480,000

The partners agreed to dissolve the partnership after selling the other assets for P200,000.
Upon dissolution of the partnership, AA should have received

A. P0
B. P48,000
C. P72,000
D. P84,000
(Dayag, 2015)

126. Larry, Marsha, and Natalie are partners in accompany that is being liquidated. They share
profits and losses 55 percent, 20 percent, and 25 percent, respectively. When the liquidation
begins, they have capital account balances of P108,000, P62,000 and P56,000,
respectively. The partnership just sold equipment with a historical cost and accumulated
depreciation of P25,000 and P18,000, respectively for P10,000. What is the balance in
Marsha’s capital account after the transaction is completed?

A. P62,000
B. P61,400
C. P62,600
D. P65,000
(Dayag, 2015)

127.Partner A first contributed P50,000 of capital into an existing partnership on March 1,


2015. On June 1, 2015, the partner contributed another P20,000. Withdrawals in excess of
P10,000 are charged to the partner’s capital account. The annual weighted-average capital
balance is

A. P62,000
B. P51,667
C. P60,000
D. P48,333

(Dayag, 2015)
128. (1) All assets contributed to the partnership are recorded by the partner at their agreed
values.
(2) All liabilities that the partnership assumes are recorded at their net present values.

A. Only the first statement is correct


B. Only the second statement is correct
C. Both statements are correct
D. Both statements are incorrect
(Dayag, 2015)

129. If a partnership has only non-cash assets, all liabilities have been properly
disbursed, and no additional liquidation expenses are expected, the maximum potential
loss to the partnership in the liquidation process is:
A. The fair market value of the non-cash assets
B. The book value of the non-cash assets
C. The estimated proceeds from the sale of the assets less the book value of the
non-cash assets.
D. None of the above.
(RESA Pre-Board July 2017)

130. In partnership,

A. Management consists of the board of directors


B. Profits are always divided equally among partners
C. Dissolution results when a partner leaves the partnership
D. No partner is liable for more than a proportion of the company’s debt
(RESA Pre-Board April 2016)

131. Mr. MAC is admitted into the partnership of Do and Nald by investing cash
equivalent to ¼ of their capital. Which of the following is true after the admission of
Mr. MAC?

A. Assets of the partnership will increase


B. Total partner’s equity remain the same
C. Do and Nald capital decreased by ¼.
D. Assets of the partnership will remain the same
(RESA Pre-Board July 2017)

132. Under the bonus method, when a new partner is admitted to the partnership, the
total capital of the new partnership is equal to:

A. The book value of the previous partnership plus the fair market value of the
consideration paid to the existing partnership by the incoming partner.
B. The book value of the previous partnership plus any necessary asset write-ups from
book value to market value plus the fair market value of the consideration paid to the
existing partnership by the incoming partner.
C. The book value of the previous partnership minus any asset write downs from book to
market value plus the fair market value of the consideration paid to the existing
partnership by the incoming partner.
D. The fair market value of the new partnership as implied by the value of the incoming
partner’s consideration in exchange for an ownership percentage in the new
partnership.
(RESA Pre-Board July 2017)
133. Which of the following statements is correct regarding a partner’s debit capital
balances?

A. The partner should make contributions to reduce the debit balance to whatever extent
possible.
B. If contributions are not possible, the other partners with credit capital balances will be
allocated a portions of the debit balance based on their proportionate profit-and-loss-
sharing percentages.
C. Partners who absorb another’s debit capital balance have a legal claim against the
deficient partner.
D. All of these statements are correct.
(RESA Pre-Board July 2017)

134. Following is the balance sheet of the ABCD Partnership at March 31, 2018, when
the partnership is to be liquidated:

Cash P 6, 000 Liabilities P 12, 400


Other Assets 126, 000 A, Loan 12, 000
B, Loan 14, 400
D, Loan 9, 600
A, Capital – 25% 16, 200
B, Capital – 25% 12, 000
C, Capital – 25% 37, 700
D, Capital – 25% 17, 700

During the month of April 2018, assets having a book value of P 18, 000 are sold at a loss of
P 2, 400. Liquidation expenses of P 600 are paid as well as P 7, 200 of the liabilities. Of the
liabilities shown in the balance sheet, P 240 represents salary payable to D and P 160 represents
salary payable to C.

On April 30, 2018 cash to be distributed to A, B, C and D as follows:

A B C D
A. P 0 P 0 P 0 P 9, 000
B. P 1, 950 P 1, 950 P 1, 950 P 1, 950
C. P 0 P 0 P 0 P 1, 950
D. P 0 P 0 P 9, 000 P 0
(RESA Pre-Board July 2017)
135. Cheryl is the manager of a local store. She is also a partner in the company and she
receives a bonus as part of the profit and loss allocation. Cheryl’s bonus is based on the
increase in revenues recorded during the period. The bonus arrangement is that Cheryl
receives 1 percent of net income for every full percentage point growth for revenues in
excess of a 5 percent revenue growth. During the most recent period, revenues grew
from P500, 000 to P540, 000 and net income grew from P 98, 000 to P 120, 000. How
much bonus does Cheryl receive for this period?

A. P 1, 100
B. P 3, 600
C. P 2, 000
D. P 6, 000
(RESA Pre-Board July 2017)

Use the following information for questions 8 and 9:

Cleary, Wasser, and Nolan formed a partnership on January 1, 20x4, with investments of
P 100, 000, P 150, 000, and P 200, 000, respectively. For division of income, they agreed
to (1) interest of 10% of the beginning capital balance each year, (2) annual compensation
of P 10, 000 to Wasser and (3) sharing the remainder of the income or loss in a ratio of
20% for Cleary and 40% each for Wasser and Nolan. Net income was P 150, 000 in 20x4
and P 180, 000 in 20x5. Each partner withdrew P 1, 000 for personal use every month
during 20x4 and 20x5.

136. What was Wasser’s share of income for 20x4?

A. P 63, 000
B. P 53, 000
C. P 58, 000
D. P 29, 000
E. P 51, 000
(RESA Pre-Board April 2016)

137. What was Wasser’s capital balance at the end of 20x5?


A. P 201, 000
B. P 263, 520
C. P 264, 540
D. P 304, 040
E. P 313, 780
(RESA Pre-Board April 2016)
138. XX, YY, and ZZ, a partnership formed on January 1, 2018 had the following initial
investment:
XX ………………………………………P 170, 000
YY ………………………………………. 255, 000
ZZ ………………………………………. 382, 500

The partnership agreement states that the profits and losses are to be shared equally by the
partners after consideration is made for the following:
- Salaries allowed to partners: P102, 000 for XX, P81, 600 for YY, and P61, 200 for ZZ.
- Average partners’ capital balances during the year shall be allowed 10%.

Additional information:
- On June 30, 2018, XX invested an additional P102, 000.
- ZZ withdrew P119, 000 from the partnership on September 30, 2018.
- Share the remaining partnership profit was P 8, 500 for each partner.

The total partnership capital on December 31, 2018 was:


A. P 688, 500
B. P 1, 141, 550
C. P 816, 000
D. d. P 1, 143, 675
(RESA Pre-Board July 2017)

139. At the time of partnership liquidation, which credits shall be settled first?

A. Those amount owing to third persons.


B. Those amount owing to partners other than capital contribution and share in profit.
C. Those amount owing to partners with respect to capital contribution.
D. Those amount owing to partners with respect to share in profit.
(CPAR Reviewer, 2017)
140. How should the net profit or net loss of the partnership be divided among the
partners, whether capitalist or industrial?

A. In accordance with their capital contribution ratio.


B. In accordance with just and equitable sharing taking into account the circumstances of
the partnership.
C. Equally
D. In accordance with the partnership agreement.
(CPAR Reviewer, 2017)
141. At the date of partnership formation of a partnership, the amount credited to A’s
capital is less than the fair value of the property contributed. Which is the most valid
reason?

A. The property contributed by A is impaired.


B. The property contributed by A has been subjected to positive asset revaluation.
C. Bonus has been given by partner A to the other partners.
D. Goodwill arising from partnership formation has been recognized.
(CPAR Reviewer, 2017)

142. When a new partner is admitted to an existing partnership through the purchase of
a portion of existing interest of an incumbent partner, which statement is correct?

A. The total capital of the old and new partnership will be the same.
B. The partnership will recognize gain or loss on the difference between the amount paid
and capital transferred.
C. Goodwill may be recognized by virtue of the admission.
D. There will be increase in the total assets of the partnership equivalent to the amount
paid by the newly admitted partner.
(CPAR Reviewer, 2017)

143. At the time of retirement, a retiring partner receives more than the amount of his
capital contribution while the remaining partners capital increase after the retirement.
Which of the following is most valid reason?

A. Goodwill during retirement is recognized.


B. Asset revaluation is recognized.
C. Bonus is given by retiring partner to remaining partners.
D. Bonus is given by the remaining partners to retiring partner.
(CPAR Reviewer, 2017)

144. On January 1, 2017, Toni, Abbie and JM entered into articles of co-partnership for
the operation of TAJ computer shop. Toni contributed investment property with assessed
value of P1,700,000 subject to mortgage payable of P500,000 to be assumed by the
partnership. Abbie contributed computer equipment with cost of P600,000 with
accumulated depreciation of P200,000. The fair market value of the computer equipment
is P300,000.
On January 2, 2017, the partnership was able to sell the investment property for
P2,000,000. How much cash shall be contributed by JM if the articles of co-partnership
provide that Toni will have 60% interest in the partnership?

A. 500,000
B. 700,000
C. 800,000
D. 600,000

(CPAR Reviewer, 2017)

145. On January 1, 2017, Yazzi, Angel and Nadine organized YAN partnership by
investing P5M, 2M and P3M for capital interest ratio of 4:5:1 respectively. Nadine has
been appointed as managing partner. During year 2017, YAN partnership reported net
income of P3,000,000. Their profit/loss distribution and drawing agreement are presented
below:

• 20% interest on beginning capital


• P10,000, P20,000 and P50,000 monthly salary, respectively
• 25% bonus of net income after interest and salary to managing partner
• The remainder will be divided equally among the partners.
• The partners must withdraw at the end of the year 50% of their share in net income for
the period.

What is the capital balance of Nadine on December 31, 2017?

A. 1,410,000
B. 3,410,000
C. 1,610,000
D. 3,610,000
(CPAR Reviewer, 2017)
146. On December 31, 2017, the capital balance of partners Cristy, Paula and Ara of
CPA Partnership are P1M, P3M and P6M, respectively with profit or loss agreement ratio
of 4:1:5. On January 1, 2018, Cristy decided to retire and received P400,000 from the
partnership.

If the assets of the partnership are not properly valued at the time of retirement, how much
is the capital balance of Paula after the retirement of Cristy?

A. 2,900,000
B. 2,850,000
C. 3,100,000
D. 3,150,000
(CPAR Reviewer, 2017)
Numbers 147 and 148
On December 31, 2017, the Statement of Financial Position of DEF with profit or loss ratio
of 4:1:5 is presented below:

Cash 2M Liability to third person 4M


Noncash asset 8M D, capital 3.5M
E, capital 1.5M
F, capital 1M

On January 31, 2018, DEF partnership has been subjected to installment liquidation. As of
January 31, 2018, the following data concerning liquidation are provided:

➢ Noncash asset with book value of P6M has been sold at a loss of P2M.
➢ Liquidation expense amounting to P400,000 has been incurred for the month of
January.
➢ P600,000 cash has been withheld for future liquidation expense.
➢ P3M liability has been paid.

147. What is F’s share in the maximum possible loss on January 31, 2018?

A. 1,300,000
B. 1,000,000
C. 1,500,000
D. 500,000

148. What is the amount received by E on January 31, 2018?

A. 300,000
B. 700,000
C. 1,000,000
D. 0
149. On December 1, 2011, EE and FF formed a partnership, agreeing to share for profits and
losses in the ratio of 2:3, respectively. EE invested a parcel of
land that cost him P25,000. FF invested P30,000 cash. The land was sold for
P50,000 on the same date, three hours after formation of the partnership.
How much should be the capital balance of EE right after formation?

A. P25,000
B. 30,000
C. 20,000
D. 50,000

150. MM, NN, and OO are partners with capital balances on December 31
2011 of P300,000, P300,000 and P200,000, respectively. Profits are shared
equally. OO wishes to withdraw and it is agreed that OO is to take certain
equipment with second-hand value of P50,000 and a note for the balance
of OO's interest. The equipment are carried on the books at P65,000. Brand
new equipment may cost P80,000. Compute for: (1) OO's acquisition of
the second-hand equipment will result to reduction in capital; (2) the value
of the note that will OO get from the partnership's liquidation.

A. (1) P15.000 each for MM and NN,(2) P150,000.


B. (1) P5,000 each for MM, NN and OO,(2) P145,000
C. (1) P5,000 each for MM. NN and OO,(2) P195,000
D. (1) P7,500 each for MM and NN,(2) P145,000.

151. JJ and KK are partners who share profits and losses in the ratio of 60%: 40%
respectively. JJ's salary is P60,000 and P30,000 for KK. The partners are also
paid interest on their averdge capital balances. In 2011, JJ received P30.000
of interest and KK, P12,000. The profit and loss allocation is determined
after deductions for the salary and interest payments. If KK's share in the
residual income (income after deducting salaries and interest) was P60,000
in 2011,

What was the total partnership income?


A. P192,000
B. 345,000
C. P282,000
D. 387,000

152. Lancelot is trying to decide whether to accept a salary of P40.000 or a


salary of P25.000 plus a bonus of 10% of net income after salary and bonus
as a means of allocating profit among the partners. Salaries traceable to
the other partners are estimated to be P100,000. What amount of income
would be necessary so that Lancelot would consider the choices to be
equal

A. P165,000
B 290,000
C. P265,000
D. 305,000

153. Merlin, a partner in the Camelot Partnership, has a 30% participation in


partnership profits and losses. Merlin's capital account has a net decrease
of P.200.000 during the calendar year 2011. During 2011, Merlin withdrew
P2.600,000 (charged against his capital account) and contributed property
valued at P500,000 to the partnership. What was the net income of the
Camelot Partnership for year 2011?

A. P3,000,000
B. 4,666,667
C. P 7,000,000
D. 11,000,000

154. The partnership agreement of XX, YY & ZZ provides for the


allocation of net income in the following order:

First, XX is to receive 10% of net income up to P200.000 and 20% over P200,000.
Second, YY and ZZ each are to receive 5% of the remain income over P300,000
The balance of income is to be allocated equally among the three partners.
The partnership's 2011 net income was P500,000 before any allocations to
partners. What amount should be allocated to XX?

A. P202,000
B. 216,000
C. P206,000
D. 220,000

155. On April 30, 2011, XX, YY and ZZ formed a partnership by combining their
separate business proprietorships. XX contributed cash of P75,000. YY
contributed property with a P54,000 carrying amount, a P60,000 original
cost, and P120,000 fair value. The partnership accepted responsibility for
the P52,500 mortgage attached to the property. ZZ contributed equipment
with a P45,000 carrying amount, a P112,500 original cost, and P82,500 fair
value. The partnership agreement specifies that profits and losses are to
be shared equally but is silent regarding capital contributions. Which
partner has the largest April 30, 2011, capital balance?

A. XX
B. YY
C. ZZ
D. All capital account balances are equal

156. The Partnership has the following accounting amounts:

(1) Sales P70,000


(2) Cost of Goods Sold P40,000
(3) Operating Expenses P10,000
(4) Salary allocations to partners P13,000
(5) Interest paid to banks P2,000
(6) Partners' withdrawals P8,000
The partnership net income (loss) is:

A. P20,000
B 18,000
C. P 5,000
157. The capital accounts of the partnership of NN, vv, and JJ on lune
are presented below with their respective profif and loss ratios: (P139,200 1/2 , 208,800 1/3 ,
96,000 1/6) On June 1, 2011, LL is admitted to the partnership when LL purchased, for
P132,000, a proportionate interest from NN and JJ in the net assets and
profits of the partnership. As a result of a transaction LL acquired a
fifth interest in the net assets and profits of the firm. What is the combined gain realized by
NN and JJ upon.the sale of a portion of their interest in
the partnership to LL?

A. P 0
B. 43,200
C P62,400
D. 82,000

158. On January 31, 2011, partners of Lon, Mac & Nan, LLP, had the following
loan and capital account balances (after closing entries for January):

Loan receivable from Lon 20,000 dr


Loan payable to Nan 60,000 cr
Lon, capital 30,000 dr
Mac, capital 120,000 dr
Nan, capital 70.000cr

The partnership's income sharing ratio was Lon, 50%; Mac, 20%, and Nan,
30%. On January 31, 2011, Ole was admitted to the partnership for a 20%
interest in total capital of the partnership in exchange for an investment
of P40,000 cash. Prior to Ole's admission, the existing partners agreed to
increase the carrying amount of the partnership's inventories to current
fair value, a P60,000 increase. The capital account to be credited to Ole:

A. P60,000
B. P40,000
C. P52,000
D. P46,000

159. Which of the following transactions will not affect the total equity of the partnership?

A. Recognition of impairment loss in case of admission of a new partner


B. Withdrawal of a partner
C. Admission of a new partner by purchase of existing partner’s interest below its
book value
D. Retirement of an existing partner with payment of above the book value of such
interest
(CPAR Final Pre-board Examination May 2017)

160. A, B, and C are partners with average capital balances during 2017 of P472,500, P238,650
and P162,350; respectively. The partners receive 10% interest on their average capital
balances; after deducting salaries of P122,325 to A and P82,625 to C, the residual profit or loss
is divided equally.

In 2017, the partnership had net loss of P125,624 before interest and salaries to partners.
What amount should A and C capital change respectively?

A. P40,844 decrease and P31,237 decrease


B. P30,267 increase and P40,448 decrease
C. P29,476 increase and P17,536 increase
D. P28,358 increase and P32,458 increase
(CPAR Final Pre-board Examination May 2017)

161. A partner was admitted in an existing partnership through investment of cash equivalent to
¼ of the new capitalization. If the capital balance of the old partners increases, what is the most
valid reason under Philippine GAAP?

A. Asset revaluation of existing partnership’s assets


B. Impairment loss of existing partnership’s assets
C. Recognition of goodwill of existing partnership
D. Receipt of bonus from the new partner
(CPAR Final Pre-board Examination May 2017)

162. Which of the following statements concerning the formation of partnership business is
correct?

A. PFRS allows recognition of goodwill arising from the formation of partnership


B. The juridical personality of the partnership arises from the issuance of
certification of registration
C. The parties may become partners only upon contribution of money or property but
not of industry or service
D. The capital to be credited to each partner upon formation may not be the amount
actually contributed by each partner
(CPAR Final Pre-board Examination May 2017)

163. Regina, Jessica and Nataly entered into a contract of partnership with a total capital
contribution of P5,000. The parties failed to register its articles of co-partnership with the
Securities and Exchange Commission. Which of the following statements is correct?

A. The contract of partnership is void because the law provides that when the capital
contribution is at least P3,000 it must be registered with Securities and Exchange
Commission
B. The contract of partnership will bind third persons
C. The contract of partnership remains to be valid
D. The partnership does not obtain juridical personality for failure to register with
Securities and Exchange Commission.
(CPAR Final Pre-board Examination May 2017)

164. In the absence of agreement as to distribution of profit, how shall the partnership profit be
distributed to the partners?

A. The industrial partner shall receive a share equivalent to the least share of a
capitalist partner while the capitalist partners shall share based on capital
contribution ratio.
B. The industrial partner shall receive a just and equitable share and the remainder
shall be distributed to the capitalist partners on the basis of capital contribution
ratio
C. The profit shall be distributed on the basis of loss contribution ratio which may
have been agreed upon by the partners
D. The profit shall be distributed equally to all partners including the industrial
partner

(CPAR Final Pre-board Examination May 2017)

165. On July 1, 2016, Anne, Bianca and Carla formed a business partnership to be operated as
an advertising agency. Anne contributed P10M cash while Bianca shall have a capital credit
of P6M upon receipt of bonus of P1M from Anne based on the provision in Articles of Co-
Partnership. The terms of the agreement provide that Anne and Bianca shall have a combined
40% capital interest in the newly formed partnership. What is the capital contribution made by
Carla to the partnership?

A. P24,000,000
B. P22,500,000
C. P25,000,000
D. P32,000,000
(CPAR Final Pre-board Examination May 2017)

166. On January 1, 2017, Angel, Bea and Colleen formed ABC & Co., a general professional
partnership for the exercise of their common profession. Angel contributed a building with a
cost of P5M and accumulated depreciation of P4M. Based on the city assessor’s records, the
building has an assessed value of P2M. The building has an annotated mortgage payable
amount to P500,000 to be assumed by the partnership.
On the other hand, Bea contributed 10,000 shares of stocks with par value of P200/share
and prevailing quoted price of P300/share. On January 2, 2017, the building contributed by
Angel was sold for P5.5M. If Colleen wants to have 20% capital interest in the newly
formed partnership, how much cash shall be contributed by her?

A. P875,000
B. P1,125,000
C. P2,125,000
D. P2,000,000
(CPAR Final Pre-board Examination May 2017)

167. On January 1, 2014, AB and QR agreed to form a partnership. The following are their
assets and liabilities:
Accounts AB QR
Cash 136,000 76,000
Accounts Receivable 88,000 48,000
Inventories 304,000 364,000
Machinery 480,000 440,000
Accounts Payable 216,000 144,000
Notes Payable 140,000 60,000

AB decided to pay off his notes payable from his personal assets. It was also agreed that
QR inventories were overstated by P24,000 and AB machinery was over depreciated by
P20,000. QR is to invest/withdraw cash in order to receive a capital credit that is 20% more
than AB’s total net investment in the partnership.

How much cash will be presented in the partnership’s statement of financial position?

A. 486,400
B. 546,300
C. 250,400
D. 640,300
(CPAR Final Pre-board Examination May 2017)

168. On December 1, 2014, MG and AN are combining their separate businesses to form a
partnership. Cash and noncash assets are to be contributed. The noncash assets to be
contributed and the liabilities to be assumed are as follows:

MG AN
Book value Fair value Book value Fair value
Accounts Receivable 250,000 262,500 200,000 195,000
Inventory 400,000 450,000 200,000 207,500
PPE 1,000,000 912,500 862,500 822,500
Accounts Payable 150,000 150,000 112,500 112,500

MG and AN are to invest equal amount of cash such that the contribution of MG would be 10%
more than the investment of AN. What is the amount of cash presented on the partnership’s
statement of Financial Position on December 1, 2014?

A. 5,025,000
B. 5,500,000
C. 5,750,000
D. 4,950,000
(CPAR Final Pre-board Examination May 2017)

169. After the admission of a new partner, the total partnership capital increased by the fair
value of the new partner’s net contributions to the partnership. The admission was accounted
for
A. Under the goodwill method
B. Under the bonus method
C. As a purchase of interest
D. As an investment in the partnership
(Millan, 2016)

170. If a new partner acquires a partnership interest directly from the partners rather than from
the partnership itself,
A. No entry is required.
B. The partnership should be revalued.
C. The existing partners’ capital accounts should be reduced and the new partner’s
account increased.
D. The partnership has undergone a quasi-reorganization.
(Punzalan, 2015)

171. Which of the following is not a characteristic of a partnership?


A. Limited liability
B. Limited life
C. Mutual agency
D. Ease of formation
(Punzalan, 2015)

172. Abel and Carr formed a partnership and agreed to divide initial capital equally, even though
Abel contributed P100,000 and Carr contributed P84,000 in identifiable assets. Under the
bonus approach to adjust the capital accounts, Carr’s unidentifiable assets should be debited
for
A. 46,000
B. 16,000
C. 8,000
D. 0
(Punzalan, 2015)

173. Alder, Benson and Carl are capitalist partners and Denver, an industrial partner. The
partnership reported a net loss of P100, 000. How much is the share of Denver in the reported
net loss?
A. 0
B. 10,000
C. 25,000
D. 100,000
(Punzalan, 2015)

174. XYZ Partnership provided for the following in their distribution of profits and losses:
First: X to receive 10 % of net income up to P100,000 and 20% of the amount in excess
thereof.
Then: Y and Z are each to receive 5% of the remaining income in excess of P150,000
after X’s share.
Finally: The balance is to be distributed equally to the three partners.

If the partnership earned a net income of P250,000, what is the total share of Partner X?

A. 100,000
B. 108,000
C. 110,000
D. 130,000
(Punzalan, 2015)

175. After incurring losses resulting from very unprofitable operations, the Goh Kong Wei
Partnership decided to liquidate when the partners’ capital balances were:

Goh, Capital (40%) P80,000


Kong, Capital (40%) 130,000
Wei, Capital (20%) 96,000

The noncash assets were sold in installment. Available cash were distributed to partners in
every sale of noncash assets. After the second sale of noncash assets, the partners received
the same amount of cash in the distribution. And from the third sale of noncash assets, cash
available for distribution amounts to P28,000, and unsold noncash assets has a book value
of P12,500. Using cash priority program, what amount did Wei receive in the third
installment of cash?

A. 11,600
B. 8,000
C. 5,600
D. 0
(Punzalan, 2015)

176. The condensed balance sheet of Adams & Gray, a partnership, at December 31, 2014,
follows:

Current assets P250,000


Equipment (net) 30,000
Total assets P280,000

Liabilities P20,000
Adams, Capital 160,000
Gray, Capital 100,000
Total liabilities and capital P280,000

On December 31, 2014, the fair values of the assets and liabilities were appraised at
P240,000 and P20,000, respectively, by an independent appraiser. On January 2, 2015, the
partnership was incorporated and 1,000 shares of P5 par value common stock were issued.
Immediately after the incorporation, what amount should the new corporation report as
additional paid in capital?
A. 270,000
B. 260,000
C. 215,000
D. 0
(Punzalan, 2015)

177. Partner Morgan is personally insolvent, owing P600,000. Personal assets will only bring
P200,000 when liquidated. At the same time, Morgan has a credit balance of P120,000. The
capital amounts of the other partners total a balance of P250,000. Under the doctrine of
marshalling of assets, how much the personal creditors of Morgan can collect?
A. 120,000
B. 200,000
C. 320,000
D. 570,000
(Punzalan, 2015)

178. The partnership agreement of Reid and Simm provides that 10% per year is to be credited
to each partner on the basis of weighted-average capital balances. A summary of Simm’s
capital account for the year-ended December 31, 2014, is as follows:

Balance, January 1 P140, 000


Additional Investment, July 1 40, 000
Withdrawal, August 1 (15, 000)
Balance, December 31 165, 000

What amount of interest should be credited on Simm’s capital account for 2014?
A. 15,250
B. 15,375
C. 16,500
D. 17,250
(Punzalan, 2015)

179. The fact that salaries paid to partners are not a component of partnership income is
indicative of
A. A departure from generally accepted accounting principles
B. Being characteristic of the entity theory
C. Being characteristic of the proprietary theory
D. Why partnerships are characterized by unlimited liability
(Punzalan, 2016)
180. The doctrine of marshalling of assets

A. Is applicable only if the partnership is insolvent


B. Allows partners to first contribute personal assets to unsatisfied partnership creditors
C. Is applicable if either the partnership is insolvent or individual partners are insolvent
D. Amount owed to personal creditors and to partnership for debit capital balances are
shared proportionately from the personal assets of the partners
(Punzalan, 2016)

181. If goodwill is traceable to the incoming partner, the new partner's capital balance equals

A. the fair market value of consideration paid by the incoming partner


B. the book value of the older partnership divided by the existing partners' ownership
percentage in the new partnership minus the book value of the old partnership.
C. incoming partner's ownership percentage multiplied by the capital of the new
partnership
D. none of the above.

(Guerrero, 2014)

AY and AN are partners who have the agreement to share profit and loss in the following manner:

AY AN
Annual salaries 261,000 259,000
Interest on average balances 5% 10%
Bonus (based on net income after salaries and interest) 10%
Remainder 50% 50%

During the year ended December 31, 2014, the partnership generated a profit of P575,000 before
any deductions. AY’s and AN’s average capital balances for the year are P600,000 and P300,000,
respectively. Income is distributed to the partners only as far as it is available.

182. How much is the total share of AN in the net income for the year ended 2014?

A. P286,500 C. P288,500
B. P287,500 D. P295,665
(RESA, 2014)
On January 1, 2014, L, M, and N formed a partnership with capital contributions of P625,000;
P750,000; and P937,500, respectively. The partners agreed that profit and loss would be allocated
as follows: P75,000 salary to each partner, 3% interest on initial capital contributions, the
remainder divided in the ratio 2:4:4, respectively to L, M, and N. The partnership generated income
amounting to P375,000 for the year 2014. During 2014, the following partnership errors were
discovered before the distribution of profit:

• In 2014, a purchase of piece of equipment costing P50,000 was expensed. The equipment has
an estimated life of ten years with equal service potential each year.
• On December 31, 2014, ending inventory was understated by P50,000.
On January 1, 2015, N decided to retire from the partnership.
183. If the balance of the capital of L after retirement amounts to P770,000, how much is the
settlement to N for his retirement?

A. P1,120,000 C. P1,085,000
B. P1,062,500 D. P1,110,875
(RESA, 2014)

184. If the balance of the capital of M after retirement amounts to P890,000, how much is the
settlement to N for his retirement?

A. P1,127,500
B. P1,090,500
C. P1,231,500
D. P1,152,500
(RESA, 2014)

On December 1, 2014, MG and AN are combining their separate businesses to form a partnership.
Cash and noncash assets are to be contributed. The noncash assets to be contributed and the
liabilities to be assumed are as follows:
MG AN
Book value Fair value Book value Fair value
Accounts Receivable 250,000 262,500 200,000 195,000
Inventory 400,000 450,000 200,000 207,500
PPE 1,000,000 912,500 862,500 822,500
Accounts Payable 150,000 150,000 112,500 112,500

MG and AN are to invest equal amount of cash such that the contribution of MG would be 10%
more than the investment of AN.
185. What is the amount of cash presented on the partnership’s statement of Financial Position
on December 1, 2014?
(RESA,2014)
A. P2,762,500
B. P2,512,500
C. P5,525,000
D. P5,025,000

On December 1, 2014, MV and CD agreed to invest equal amounts and share profits equally to
form a partnership. MV invested P3,120,000 cash and a piece of equipment. CD invested some
assets which are shown on the next page:

Book value

Accounts Receivable 400,000

Inventory 1,120,000

Machineries, net 2,240,000

Intangibles, net 920,000

The assets invested by CD are not properly valued, P32,000 of the accounts receivable are proven
uncollectible. Inventories are to be written down to P1,040,000. Included in the machineries is an
obsolete apparatus acquired for P384,000 with an accumulated depreciation balance of P336,000.
Part of the intangibles is a patent with a carrying value of P56,000 which was sued upon by a
competitor. CD unsuccessfully defended the case and the final decision of the court was released
on November 29, 2014.

186. What is the fair value of the equipment invested by MV?

A. P1,400,000 C. P1,344,000
B. P968,000 D. P1,560,000
(RESA, 2014)
The partnership of CD, AY, and GP decided to liquidate their partnership on May 31, 2013. Before
liquidating and sharing of net income, their capital balances are as follows: CD (30%) P875,000,
AY (30%) P630,000, and GP (40%) P770,000. Net income from January 1 to May 31 is P420,000.
Liabilities of the partnership amounted to P735,000 and its total assets include cash amounting to
P245,000. Unsettled liabilities are P385,000. CD invested additional cash enough to settle their
partnership’s indebtedness. AY is personally solvent, GP is personally insolvent, and CD becomes
insolvent after investing the cash needed by the partnership.

187. How much were the partnership’s non-cash sold for?


A. P157,500 C. P105,000
B. P3,080,000 D. P525,000

188. How much will CD receive as a result of their liquidation?


A. P385,000
B. 0
C. P315,000
D. P462,000
(RESA, 2014)
189. On April 30, 2016, Al, Ben, and Ces formed a partnership by combining their separate
business proprietorships. Al contributed cash of P50,000. Ben contributed property with a
P36,000 carrying amount, a P40,000 original cost, and P80,000 fair value. The partnership
accepted responsibility for the P35,000 mortgage attached to the property. Ces contributed
equipment with a P30,000 carrying amount, a P75,000 original cost, and P55,000 fair value.
The partnership agreement specifies that profits and losses are to be shared equally but is
silent regarding capital contributions. Which partner has the largest capital account balance
at April 30, 2016?
A. Al
B. Ben
C. Ces
D. All capital balances are equal
(Punzalan, 2015)

190. A partnership records a partner’s investment of assets in the business at


A. The market value of the assets invested.
B. A special value set by the partners.
C. The partner’s book value of the assets invested.
D. Any of the above, depending upon the partnership agreement.
(RPCPA 0598)
191. In the Adel-Brick partnership, Adel and Brick had a capital ratio of 3:1 and a profit and
loss ratio of 2:1, respectively. The bonus method was used to record Colter’s admittance as a
new partner. What ratio would be used to allocate, to Adel and Brick, the excess of Colter’s
contribution over the amount credited to Colter’s capital account?

A. Adel and Brick’s new relative capital ratio.


B. Adel and Brick’s new relative profit and loss ratio.
C. Adel and Brick’s old capital ratio.
D. Adel and Brick’s old profit and loss ratio.
(AICPA 0r92 T-35)
192. The final cash distribution to the partners in a partnership in liquidation should be made in
accordance with

A. Balances of the partners’ capital accounts.


B. Partners’ profit and loss sharing ratio.
C. Ratio of capital contributions made by the partners.
D. Ratio of capital contributions less withdrawals made by the partners.
(RPCPA 1081,0586)
193. K, L, and M are partners with average capital balance during 2011 of P472,500, P238,650,
and P162,350, respectively. The partners receive 10% interest on their average capital
balances; after deducting salaries of P122,325 to K and P82,625 to M, the residual profits or
loss is divided equally.

In 2011, the partnership had a net loss of P125,624 before the interest and salaries to
partners.

By what amount should K’s and M’s capital account change?


K’s Capital Account M’s Capital Account
A. P40,844 decrease P31,235 decrease
B. P28,358 increase P32,458 increase
C. P29,476 increase P17,536 increase
D. P30,267 increase P40,448 decrease
(Guerrero, 2013)
194. Prior to partnership liquidation, a schedule of possible losses is frequently prepared to
determine the amount of cash that may be safely distributed to the partners. The schedule of
possible losses

A. Consists of each partner’s capital account plus loan balance, divided by that
partner’s profit-and-loss sharing ratio.
B. Shows the successive losses necessary to eliminate the capital accounts of partners
(assuming no contribution of personal assets by partners).
C. Indicates the distribution of successive amounts of available cash to each partner.
D. Assumes contribution of personal assets by partners unless there is a substantial
presumption of personal insolvency by the partners.
(Gleim)
195. The following condensed balance sheet is presented for the partnership of Axel, Barr, and
Cain, who share profits and losses in the ratio of 4:3:3, respectively:
Cash P100,000
Other assets 300,000
Total 400,000

Liabilities P150,000
Axel, Capital 40,000
Barr, Capital 180,000
Cain, Capital 30,000
Total 400,000

The partners agreed to dissolve the partnership after selling the other asset for P200,000.
Upon dissolution of the partnership, Axel should have received

A. 0
B. 40,000
C. 60,000
D. 70,000
(Punzalan, 2015)
196. The following blance sheet is presented for the partnership of A, B, and C, who share
profits and losses in the respectively ratio of 5:3:2.
Assets Liabilities and Capital
Cash P 120,000 Liabilities P280,000
Other Assets 1,080,000 A, Capital 560,000
B, Capital 320,000
C, Capital 40,000
Total P1,200,000 Total P1,200,000

Assume that the three partners decided to liquidate the partnership. If the other assets are
sold for P800,000, how should the available cash be distributed to each partner?
A B C
A. 280,000 320,000 40,000
B. 324,000 236,000 16,000
C. 410,000 230,000 0
D. 412,000 228,000 0
(Punzalan, 2015)

197. Red, White, and Blue form a partnership on May 1, 2011. They agree that Red will
contribute office equipment with a toal fair value of P40,000; White will contribute delivery
equipment with a fair value of P80,000; and Blue will contribute cash. If Blue wants a on third
interest in the capital and profits, he should contribute cash of:
A. P40,000
B. P120,000
C. P60,000
D. P180,000
(Guerrero, 2013)

198. AK and BK decided to form a partnership on October 1, 2014. Their Statement of


Financial Position on this date were:
AK Bk
Cash 65,625.00 164,062.50
Accounts Receivable 1,487,500.00 896,875.00
Merchandise Inventory 875,000.00 885,937.50
Equipment 656,250.00 1,268,750.00
Total 3,084,375.00 3,215,625.00

Accounts Payable 459,375.00 1,159,375.00


AK, Capital 2,625,000.00
BK, Capital 2,056,250.00
Total 3,084,375.00 3,215,625.00

They agreed the following adjustments shall be made:


• Equipment of AK is underdepreciated by P87,500 and that BK is overdepreciated by
P131,250.
• Allowance for doubtful accounts is to be set up amounting to P297,500 for AK and P196,875
for BK.
• Inventories of P21,875 and P15,312.50 are worthless in the books of AK and BK
respectively.
• The partnership agreement provides for a profit and loss ratio of 70% to AK and 30% to BK.
Assuming the use of transfer of capital method, how much is the agreed capital of AK to bring
the capital balances proportionate to their profit and loss ratio.
On January 1, 2014, AB and QR agreed to form a partnership. The following are their assets and
liabilities:
Accounts AB QR
Cash 136,000 76,000
Accounts Receivable 88,000 48,000
Inventories 304,000 364,000
Machinery 480,000 440,000
Accounts Payable 216,000 144,000
Notes Payable 140,000 60,000

AB decided to pay off his notes payable from his personal assets. It was also agreed that QR
inventories were overstated by P24,000 and AB machinery was over depreciated by P20,000. QR
is to invest/withdraw cash in order to receive a capital credit that is 20% more than AB’s total net
investment in the partnership.
How much cash will be presented in the partnership’s statement of financial position?
A. 2,935,406.25
B. 2,218,125.00
C. 1,975,312.50
D. 1,258,031.25
CPAR Pre-Boards October 2017

199. On December 1, 2014, MV and CD agreed to invest equal amounts and share profits
equally to form a partnership. MV invested P3,120,000 cash and a piece of equipment. CD
invested some assets which are shown on the next page:
Book value
Accounts Receivable 400,000
Inventory 1,120,000
Machineries, net 2,240,000
Intangibles, net 920,000

The assets invested by CD are not properly valued, P32,000 of the accounts receivable are
proven uncollectible. Inventories are to be written down to P1,040,000. Included in the
machineries is an obsolete apparatus acquired for P384,000 with an accumulated depreciation
balance of P336,000. Part of the intangibles is a patent with a carrying value of P56,000 which
was sued upon by a competitor. CD unsuccessfully defended the case and the final decision of
the court was released on November 29, 2014.
A. 274,000
B. 212,000
C. 486,000
D. 374,000
CPAR Pre-Boards October 2017

200. On December 1, 2014, MG and AN are combining their separate businesses to form a
partnership. Cash and noncash assets are to be contributed. The noncash assets to be
contributed and the liabilities to be assumed are as follows:
MG AN
Book value Fair value Book value Fair value
Accounts Receivable 250,000 262,500 200,000 195,000
Inventory 400,000 450,000 200,000 207,500
PPE 1,000,000 912,500 862,500 822,500
Accounts Payable 150,000 150,000 112,500 112,500

MG and AN are to invest equal amount of cash such that the contribution of MG would be 10%
more than the investment of AN.
What is the amount of cash presented on the partnership’s statement of Financial Position on
December 1, 2014?

A. 1,344,000
B. 1,244,000
C. 3,120,000
D. 2,180,000

CPAR Pre-Boards October 2017


201. CC Partnership began operations on June 1, 2014. On that date, CY and CR have capital
credits of P175,000 and P240,000, respectively. The partnership has the following profit-
sharing plan:
a.) 10% interest on partners’ capital balances at the end of the year
b.) P60,000 and P75,000 annual salaries for CY and CR, respectively.
c.) Remaining profit will be divided to CY and CR on a 3:2 ratio, respectively.

During the year, CY invested P150,000 worth of merchandise and withdrew P40,000 cash,
while CR invested P120,000 cash. The partnership earned a profit of P266,375 during the year.
How much is CY’s capital balance at the end of 2014?
A. 5,025,000
B. 2,512,000
C. 3,215,000
D. 1,223,750
CPAR Pre-Boards October 2017
202. CC Partnership began operations on June 1, 2014. On that date, CY and CR have capital
credits of P175,000 and P240,000, respectively. The partnership has the following profit-
sharing plan:
a) 10% interest on partners’ capital balances at the end of the year
b) P60,000 and P75,000 annual salaries for CY and CR, respectively.
c) Remaining profit will be divided to CY and CR on a 3:2 ratio,
respectively.

During the year, CY invested P150,000 worth of merchandise and withdrew P40,000 cash, while
CR invested P120,000 cash. The partnership earned a profit of P266,375 during the year.
How much is CY’s capital balance at the end of 2014?
A. 266,375
B. 426,625
C. 285,000
D. 150,000
CPAR Pre-Boards October 2017

203. Cherryhill and Hace had been partners for several years, and they decided to admit Quincy
to the partnership. The accountant for the partnership believed that the dissolved partnership
and the newly formed partnership were two separate entities. What method would the
accountant have used for recording the admission of Quincy to the partnership?

A) the bonus method.


B) the equity method.
C) the goodwill method.
D) the proportionate method.
E) the cost method.
CPAR Pre-Boards October 2017

204. When the hybrid method is used to record the withdrawal of a partner, the partnership

A) revalues assets and liabilities and records goodwill to the continuing partner but not to the
withdrawing partner.
B) revalues liabilities but not assets, and no goodwill is recorded.
C) can recognize goodwill but does not revalue assets and liabilities.
D) revalues assets but not liabilities, and records goodwill to the continuing partner but not
to the withdrawing partner.
E) revalues assets and liabilities but does not record goodwill.
CPAR Pre-Boards October 2017
205. The disadvantages of the partnership form of business organization, compared to
corporations, include
A) the legal requirements for formation.
B) unlimited liability for the partners.
C) the requirement for the partnership to pay income taxes.
D) the extent of governmental regulation.
E) the complexity of operations.
CPAR Pre-Boards October 2017
206. The advantages of the partnership form of business organization, compared to
corporations, include

A) single taxation.
B) ease of raising capital.
C) mutual agency.
D) Limited liability.
E) difficulty of formation.
CPAR Pre-Boards October 2017
207. The dissolution of a partnership occurs

A) only when the partnership sells its assets and permanently closes its books.
B) only when a partner leaves the partnership.
C) at the end of each year, when income is allocated to the partners.
D) only when a new partner is admitted to the partnership.
E) when there is any change in the individuals who make up the partnership.
CPAR Pre-Boards October 2017

208. If a partner’s capital balance is credited for an amount greater than or less than the fair
value of his net contribution, the excess or deficiency is called a
A. Bonus
B. Goodwill
C. Discount
D. Premium
(Millan, 2016)
209. If the partnership agreement does not specify how income is to be allocated, profits and
loss should be allocated
A. Equally
B. In proportion to the weighted average of capital invested during the period
C. Equitably so that partners are compensated for the time and effort expended
on behalf of the partnership
D. In accordance with their capital contributions
(Millan, 2016)
210. When property other than cash is invested in a partnership, at what amount should the
noncash property be credited to the contributing partner’s capital account?

A. Fair value at the date of contribution


B. Contributing partner’s original cost
C. Assessed valuation for property tax purposes
D. Contributing partner’s tax basis
(Millan, 2016)
211. The admission of a new partner effected through purchase of interest in the partnership is

A. Recorded in the partnership books as a debit to cash or other asset and credit
to the incoming partner’s capital account
B. Recorded in the partnership books as a transfer within equity
C. Recorded in the partnership books as a transfer from equity to liability
D. Not recorded in its entirety
(Millan, 2016)

212. State the proper order of liquidation


I. Outside creditors
II. Owners’ interests
III. Inside creditors
A. I, III, II
B. I, II, III
C. III, II, I
D. II, I, III

(Millan, 2016)

213. Lancelot is trying to decide whether to accept a salary of P40,000 or a salary of P25,000
plus a bonus of 10% of net income after salary and bonus as a means of allocating profit among
the partners. Salaries traceable to the other partners are estimated to be P100,000. What amount
of income would be necessary so that Lancelot would consider the choices to be equal?
A. P 165,000
B. P 290,000
C. P 265,000
D. P 305,000
(Dayag, 2015)
214. MM, NN, OO are partners with capital balances on December 31, 2015 of P 300,000, P
300,000 and P 200,000, respectively. Profits are shared equally. OO wishes to withdraw and it
is agreed that OO is to take certain equipment with second-hand value of P 50,000 and a note
for the balance of OO’s interest. The equipment are carried on the books at P65,000. Brand
new equipment may cost P 80,000. Compute for: (1) OO’s acquisition of the second-hand
equipment that will result to reduction in capital; (2) the value of the note that will OO get
from the partnership’s liquidation.
A. (1) P15,000 each for MM and NN, (2) P150,000
B. (1) P5,000 each for MM, NN and OO, (2) P145,000
C. (1) 5,000 each for MM, NN and OO, (2) P195,000
D. (1) P7,500 each for MM and NN, (2) P145,000
(Dayag, 2015)
215. RR and XX formed a partnership and agreed to divide initial capital equally, even though
RR contributed P25,000 and XX contributed P21,000 in identifiable assets. Under the bonus
approach to adjust the capital accounts. XX’s unidentifiable assets should be debited for:
A. P 11,500
B. P 4,000
C. P 2,000
D. P 0

(Dayag, 2015)

216. A. Smith, a partner in an accounting firm, decided to withdraw from the partnership,
Smith’s share of the partnership profits and losses was 20%. Upon withdrawing from the
partnership he was paid P88,800 in final settlement for his interest. The total of the partner’s
capital accounts before recognition of partnership goodwill prior to Smith’s withdrawal was
P252,000. After his withdrawal the remaining partners’ capital accounts, excluding their share
of goodwill, totaled P192,000. The total goodwill of the firm was:
A. P 144,000
B. P 168,000
C. P 192,000
D. P 300,000

(Dayag, 2015)

217. The following condensed balance sheet is presented for the partnership of AA, BB, and
CC, who share profits and losses in the ratio of 4:3:3, respectively:

Cash P 160,000
Other assets 320,000
Total P 480,000

Liabilities P 180,000
AA, Capital 48,000
BB, Capital 216,000
CC, Capital 36,000
Total P 480,000
The partners agreed to dissolve the partnership after selling the other assets for P200,000.
Upon dissolution of the partnership, AA should have received

A. P 0
B. P 48,000
C. P 72,000
D. P 84,000

(Dayag, 2015)

218. When property other than cash is invested in a partnership, at what amount should the
noncash property be credited to the contributing partner’s capital account?
A. Fair Value at the date of recognition
B. Contributing partner’s original cost
C. Assessed valuation for property tax purposes
D. Contributing partner’s tax basis
(AICPA 0594 F-35)

219. A partnership records a partner’s investment of assets in the business at

A. The market value of the assets invested


B. A special value set by the partners
C. The partner’s book value of the assets invested
D. Any of the above, depending upon the partnership agreement
(RPCPA 0598)

220. In a partnership liquidation, the final cash distribution to the partners should be made in
accordance with the

A. Partners’ profit and loss sharing ratio


B. Balances of the partners’ capital accounts
C. Ratio of capital contributions made by the partners
D. Ratio of capital contributions less withdrawals made by the partners
(RPCPA 1079)

221. As a result of the retirement of a partner in an existing partnership, the capital balance of
the remaining partners increases. If the assets of the partnership before retirement are
properly valued, which of the following statements is true?

A. The retiring partner received less than his capital balance before retirement
B. There is partnership net loss prior to the retirement of the said partner
C. The remaining partner gives bonus to the retiring partner
D. There is impairment of existing assets recognized prior to retirement
(CPAR PREBOARD WEEK, 2017)
222. If the partnership agreement does not specify how income is to be allocated, profits
should be allocated

A. Equally
B. In proportion to the weighted-average of capital invested during the period
C. Equitably so that partners are compensated for the time and effort expended
on behalf of the partnership
D. In accordance with an established ratio
(Gleim)

223. On June 30, 2016, a partnership was formed by Mendoza and Lopez. Mendoza
contributed cash. Lopez, previously a sole proprietor contributed non-cash assets
including a realty subject to a mortgage which was assumed by the partnership. Lopez’s
capital account at June 30,2016 should be recorded at

A. The fair value of the property on June 30, 2016


B. Lopez’s carrying amount of the property on June 30, 2016
C. The fair value of the property on June 30, 2016 less the mortgage payable
D. Lopez’s carrying amount of the property on June 30, 2016 less the mortgage
payable
(BAYSA & LUPISAN, 2016)

For numbers 224 to 225 refer to the problem below:


Diaz and Esteban entered into a partnership on February 1, 2016 by investing the following
assets:

Diaz Esteban
Cash P 15,000
Merchandise Inventory P 45,000
Land 15,000
Building 65,000
Furniture and Fixtures 100,000

The agreement between Diaz and Esteban provides that profits and losses are to be divided into
40% and 60% to Diaz and Esteban respectively. The partnership is to assume the P30,000
mortgage loan on the building.
224. If Esteban is to receive a capital credit equal to his profit and loss ratio, how much cash
must he invest?

A. 77,500
B. 97,500
C. 127500
D. 172,500
(BAYSA & LUPISAN, 2016)

225. Assuming Esteban invests P50,000 cash and each partner is to be credited for the full
amount of the net assets invested, the total capital of the partnership is

A. 210,000
B. 250,000
C. 260,000
D. 290,000
(BAYSA & LUPISAN, 2016)

226. Assuming the partnership agreement provides that the partners should initially have an
equal interest in the partnership capital, what is Esteban’s capital upon partnership
formation?
A. 95,000
B. 105,000
C. 115,000
D. 125,000
(BAYSA & LUPISAN, 2016)

227. Canlas, a partner in the 3C Partnership, has a 30% participation in partnership profits and
lossess. Canlas’ capital account had a net decrease of P120,000 during the calendar year
2016. During 2016, Canlas withdrew P260,000 (charged against his capital account) and
contributed property valued at P50,000 to the partnership. What was the profit of 3C
Partnership?

A. 300,000
B. 466,667
C. 700,000
D. 1,100,000
(BAYSA & LUPISAN, 2016)

228. Profit is the difference between


A. assets and liabilities
B. the incoming cash and outgoing cash
C. the assets purchased with cash contributed by the owner and the cash spent to operate
the business
D. the assets received for goods and services and the amounts used to provide the goods
and services

(Warren 9th ed.)

229. Which of the items below is not a business organization form?

A. Entrepreneurship C. Partnership
B. Proprietorship D. Corporation
(Warren 9th ed.)

230. An entity that is organized in which ownership is divided into shares of stock is a

A. Proprietorship C. Partnership
B. Corporation D. Governmental Unit

(Warren 9th ed.)

231. Financial reports are used by

A. Management C. Inventors
B. Creditors D. All are correct
(Warren 9th ed.)

232. Which of the following is not a characteristic of a corporation?


A. Corporations are organized as a separate legal taxable entity
B. Ownership is divided into shares of stock.
C. Corporations experience an ease in obtaining large amounts of resources by issuing
stock.
D. A corporation’s resources are limited to their individual owners’ resources.

(Warren 9th ed.)

233. He refers to a partner who contributed not only money and property but also
industry to the newly formed partnership.

A. industrial partner
B. nominal partner
C. capitalist-industrial partner
D. capitalist partner
(CPAR handout, 2018)

234. It refers to a type of partnership wherein all partners are liable to the creditors pro-
rata up to the extent of personal or separate assets after the partnership’s asset are
exhausted.

A. General partnership
B. Partnership by estoppel
C. Limited partnership
D. Particular partnership
(CPAR handout, 2018)
235. Which of the following will decrease the capital balance of a partner?
A. Share in partnership profit
B. Receipt of share in revaluation surplus from a partnership property, plant and
equipment
C. Drawing made by partner
D. Advances made by a partner to the partnership
(CPAR handout, 2018)
A, B and C decided to form ABC Partnership. It was agreed that A will contribute an equipment
with assessed value of P100,000 with historical cost of P800,000 and accumulated depreciation of
P600,000. A day after the partnership formation, the equipment was sold for P300,000.

B will contribute a land and building with carrying amount of P1,200,000 and fair value of
P1,500,000. The land and building are subject to a mortgage payable amounting to P300,000 to be
assumed by the partnership. The partners agreed that B will have 60% capital interest in the
partnership. The partners also agreed that C will contribute sufficient cash to the partnership.

236. What is the total agreed capitalization of ABC Partnership?


A. 1,500,000
B. 2,000,000
C. 2,500,000
D. 3,000,000

237. What is the cash to be contributed by C in the ABC Partnership?


A. 500,000
B. 600,000
C. 700,000
D. 800,000
(CPAR handout, 2018)

238. When property other than cash is invested in a partnership, at what amount should
the noncash property be credited to the contributing partner’s capital account?

A. Contributing partner’s tax basis


B. Contributing partner’s original cost
C. Fair value at the date of contribution
D. Assessed valuation for property tax purposes
(Wiley 2014)
239. In a limited partnership, a general partner
A. Is excluded from management
B. Is not entitled to a bonus at the end of the year
C. Has limited liability for partnership debt
D. Has unlimited liability for partnership debt
(BCA & L 10e)
240. Partnership drawings are
A. Usually maintained in a separate account from the partner’s capital account
B. Equal to partner’s salaries
C. Similar to advances made to partners and are included as assets on the balance sheet
D. Not discussed in the specific contract provisions of the partnership
(FT & C 11e)
241. Which of the following is an advantage of a partnership?
A. Mutual agency
B. Limited life
C. Unlimited liability
D. None of these
(J & C 3e)
242. The profit and loss sharing ratio should be
A. In the same ratio as the percentage interest owned by each partner
B. Based on relative effort contributed to the firm by the partners
C. A weighted average of capital and effort contributions
D. Based on any formula that the partners choose
(J & C 3e)
243. Maxwell is trying to decide whether to accept a salary of $60,000 or a salary of
$25,000 plus a bonus 0f 20% of net income after the bonus as a means of allocating profit
among the partners. What amount of income would be necessary so that Maxwell would
consider the choices to be equal?
A. $35,000
B. $85,000
C. $140,000
D. $210,000
(FT & C 11e)
244. Joan a senior partner in a fashion designing firm has a share of 30% in earnings. In
19x8, she transferred to the firm, property with current fair value of P25,000 but made
capital withdrawal of P130,000. If her closing capital balance was P60,000 less than her
beginning capital balance, how much was the partnership’s net income loss in 19x8?

A. P(45,000)
B. P135,000
C. P150,000
D. P180,000
(RRCPA 0598)
245. JJ and KK are partners who share profits and losses in the ratio of 60% and 40%
respectively. JJ’s salary is P60,000 and P30,000 for KK, the partners are also paid interest
on their average capital balances. In 2012, JJ received P30,000 of interest and KK, P12,000.
The profit and loss allocation is determined after deductions for the salary and interest
payments. If KK’s share in the residual income (income after deducting salaries and
interest) was P60,000 in 2012, what was the total partnership income?

A. P192,000
B. P345,000
C. P282,000
D. P387,000
(Dayag 11)

FINISH!
A year ago, you did not know today.
You did not know how you’d make it here.
But you made it here!

By grace, you made it here! 😊


- (M.H.N)

You might also like